Exam 2

Pataasin ang iyong marka sa homework at exams ngayon gamit ang Quizwiz!

The vaginal examination is an essential component of labor assessment. It reveals whether the client is in true labor and enables the examiner to determine whether membranes have ruptured. The vaginal examination is often stressful and uncomfortable for the client and should be performed only when indicated. Match the correct step number, from 1 to 7, with each component of a vaginal examination of the laboring woman. a. After obtaining permission, gently insert the index and middle fingers into the vagina. b. Explain the findings to the client. c. Position the woman to prevent supine hypotension. d. Use sterile gloves and soluble gel for lubrication. e. Document findings and report to the health care provider. f. Cleanse the perineum and vulva, if necessary. g. Determine dilation, presenting part, status of membranes, and characteristics of amniotic fluid.

1. Step 1 -> Use sterile gloves and soluble gel for lubrication. 2. Step 2 -> Position the woman to prevent supine hypotension. 3. Step 3 -> Cleanse the perineum and vulva, if necessary. 4. Step 4 -> After obtaining permission, gently insert the index and middle fingers into the vagina. 5. Step 5 -> Determine dilation, presenting part, status of membranes, and characteristics of amniotic fluid. 6. Step 6 -> Explain the findings to the client. 7. Step 7 -> Document findings and report to the health care provider.

A woman has requested an epidural block for her pain. She is 5 cm dilated and 100% effaced. The baby is in a vertex position and is engaged. The nurse increases the woman's IV fluid for a preprocedural bolus. Before the initiation of the epidural, the woman should be informed regarding the disadvantages of an epidural block. Which concerns should the nurse share with this client? (Select all that apply.) a. Ability to move freely is limited. b. Orthostatic hypotension and dizziness may occur. c. Gastric emptying is not delayed. d. Higher body temperature may occur. e. Blood loss is not excessive.

ANS: A - Ability to move freely is limited. B - Orthostatic hypotension and dizziness may occur. D - Higher body temperature may occur. The woman's ability to move freely and to maintain control of her labor is limited, related to the use of numerous medical interventions (IV lines and electronic fetal monitoring [EFM]). Significant disadvantages of an epidural block include the occurrence of orthostatic hypotension, dizziness, sedation, and leg weakness. Women who receive an epidural block have a higher body temperature (38° C or higher), especially when labor lasts longer than 12 hours, and may result in an unnecessary neonatal workup for sepsis. An advantage of an epidural block is that blood loss is not excessive. Other advantages include the following: the woman remains alert and able to participate, good relaxation is achieved, airway reflexes remain intact, and only partial motor paralysis develops.

Which alternative approaches to relaxation have proven successful when working with the client in labor? (Select all that apply.) a. Aromatherapy b. Massage c. Hypnosis d. Cesarean birth e. Biofeedback

ANS: A - Aromatherapy B - Massage C - Hypnosis E - Biofeedback Approaches to relaxation can include neuromuscular relaxation, aromatherapy, music, massage, imagery, hypnosis, or touch relaxation. Cesarean birth is a method of delivery, not a method of relaxation.

While developing an intrapartum care plan for the client in early labor, which psychosocial factors would the nurse recognize upon the client's pain experience? (Select all that apply.) a. Culture b. Anxiety and fear c. Previous experiences with pain d. Intervention of caregivers e. Support systems

ANS: A - Culture B - Anxiety and fear C - Previous experiences with pain E - Support systems Culture: A woman's sociocultural roots influence how she perceives, interprets, and responds to pain during childbirth. Some cultures encourage loud and vigorous expressions of pain, whereas others value self-control. The nurse should avoid praising some behaviors (stoicism) while belittling others (noisy expression). Anxiety and fear: Extreme anxiety and fear magnify the sensitivity to pain and impair a woman's ability to tolerate it. Anxiety and fear increase muscle tension in the pelvic area, which counters the expulsive forces of uterine contractions and pushing efforts. Previous experiences with pain: Fear and withdrawal are a natural response to pain during labor. Learning about these normal sensations ahead of time helps a woman suppress her natural reactions of fear regarding the impending birth. If a woman previously had a long and difficult labor, she is likely to be anxious. She may also have learned ways to cope and may use these skills to adapt to the present labor experience. Support systems: An anxious partner is less able to provide help and support to a woman during labor. A woman's family and friends can be an important source of support if they convey realistic and positive information about labor and delivery. Although the intervention of caregivers may be necessary for the well-being of the woman and her fetus, some interventions add discomfort to the natural pain of labor (i.e., fetal monitor straps, IV lines)

Congenital disorders refer to those conditions that are present at birth. These disorders may be inherited and caused by environmental factors or maternal malnutrition. Toxic exposures have the greatest effect on development between 15 and 60 days of gestation. For the nurse to be able to conduct a complete assessment of the newly pregnant client, he or she should be knowledgeable regarding known human teratogens. Which substances might be considered a teratogen? (Select all that apply.) a. Cytomegalovirus (CMV) b. Ionizing radiation c. Hypothermia d. Carbamazepine e. Lead

ANS: A - Cytomegalovirus (CMV) B - Ionizing radiation D - Carbamazepine E - Lead Exposure to radiation and a number of infections may result in profound congenital deformities. These include but are not limited to varicella, rubella, syphilis, parvovirus, CMV, and toxoplasmosis. Certain maternal conditions such as diabetes and phenylketonuria (PKU) may also affect organs and other parts of the embryo during this developmental period. Drugs such as antiseizure medications (e.g., carbamazepine) and some antibiotics, as well as chemicals including lead, mercury, tobacco, and alcohol, may also result in structural and functional abnormalities.

A woman is in for a routine prenatal checkup. The nurse is assessing her urine for glycosuria and proteinuria. Which findings are considered normal? (Select all that apply.) a. Dipstick assessment of trace to +1 b. <300 mg/24 hours c. Dipstick assessment of +2 glucose d. >300 mg/24 hours e. Albumin < 30 mg/24 hours

ANS: A - Dipstick assessment of trace to +1 B - <300 mg/24 hours E - Albumin < 30 mg/24 hours Small amounts of protein in the urine are acceptable during pregnancy. The presence of protein in greater amounts may indicate renal problems. A dipstick assessment of +2 and proteinuria >300 mg/24 hours, and albuminuria greater than 30 mg/24 hours are excessive and should be further evaluated.

Pregnancy is a hypercoagulable state in which women are at a fivefold to sixfold increased risk for thromboembolic disease. The tendency for blood to clot is greater, attributable to an increase in various clotting factors. Which of these come into play during pregnancy? (Select all that apply.) a. Factor VII b. Factor VIII c. Factor IX d. Factor XIII e. Fibrinogen

ANS: A - Factor VII B - Factor VIII C - Factor IX E - Fibrinogen Factors VII, VIII, IX, X, and fibrinogen increase in pregnancy. Factors that inhibit coagulation decrease. Fibrinolytic activity (dissolving of a clot) is depressed during pregnancy and the early postpartum period to protect the women from postpartum hemorrhage.

Relating to the fetal circulatory system, which special characteristics allow the fetus to obtain sufficient oxygen from the maternal blood? (Select all that apply.) a. Fetal hemoglobin (Hb) carries 20% to 30% more oxygen than maternal Hb. b. Fetal Hb carries 40% to 50% more oxygen than maternal Hb. c. Hb concentration is 50% higher than that of the mother. d. Fetal heart rate is 110 to 160 beats per minute. e. Fetal heart rate is 160 to 200 beats per minute.

ANS: A - Fetal hemoglobin (Hb) carries 20% to 30% more oxygen than maternal Hb. C - Hb concentration is 50% higher than that of the mother. D - Fetal heart rate is 110 to 160 beats per minute. The following three special characteristics enable the fetus to obtain sufficient oxygen from maternal blood: (1) the fetal Hb carries 20% to 30% more oxygen; (2) the concentration is 50% higher than that of the mother; and (3) the fetal heart rate is 110 to 160 beats per minute, a cardiac output that is higher than that of an adult

The diagnosis of pregnancy is based on which positive signs of pregnancy? (Select all that apply.) a. Identification of fetal heartbeat b. Palpation of fetal outline c. Visualization of the fetus d. Verification of fetal movement e. Positive hCG test

ANS: A - Identification of fetal heartbeat C - Visualization of the fetus D - Verification of fetal movement Identification of a fetal heartbeat, the visualization of the fetus, and verification of fetal movement are all positive, objective signs of pregnancy. Palpation of fetal outline and positive hCG test are probable signs of pregnancy. A tumor also can be palpated. Medication and tumors may lead to false-positive results on pregnancy tests.

Which changes take place in the woman's reproductive system, days or even weeks before the commencement of labor? (Select all that apply.) a. Lightening b. Exhaustion c. Bloody show d. Rupture of membranes e. Decreased fetal movement

ANS: A - Lightening C - Bloody show D - Rupture of membranes Signs that precede labor may include lightening, urinary frequency, backache, weight loss, surge of energy, bloody show, and rupture of membranes. Many women experience a burst of energy before labor. A decrease in fetal movement is an ominous sign that does not always correlate with labor.

In assessing the immediate condition of the newborn after birth, a sample of cord blood may be a useful adjunct to the Apgar score. Cord blood is then tested for pH, carbon dioxide, oxygen, and base deficit or excess. Which clinical situation warrants this additional testing? (Select all that apply.) a. Low 5-minute Apgar score b. Intrauterine growth restriction (IUGR) c. Maternal thyroid disease d. Intrapartum fever e. Vacuum extraction

ANS: A - Low 5-minute Apgar score B - Intrauterine growth restriction (IUGR) C - Maternal thyroid disease D - Intrapartum fever The American College of Obstetricians and Gynecologists (ACOG) suggests obtaining cord blood values in all of these clinical situations except for vacuum extractions deliveries. Cord blood gases should also be performed for multifetal pregnancies or abnormal FHR tracings. Samples can be drawn from both the umbilical artery and the umbilical vein. Results may indicate that fetal compromise has occurred

Emergency conditions during labor that would require immediate nursing intervention can arise with startling speed. Which situations are examples of such an emergency? (Select all that apply.) a. Nonreassuring or abnormal FHR pattern b. Inadequate uterine relaxation c. Vaginal bleeding d. Prolonged second stage e. Prolapse of the cord

ANS: A - Nonreassuring or abnormal FHR pattern B - Inadequate uterine relaxation C - Vaginal bleeding E - Prolapse of the cord A nonreassuring or abnormal FHR pattern, inadequate uterine relaxation, vaginal bleeding, infection, and cord prolapse all constitute an emergency during labor that requires immediate nursing intervention. A prolonged second stage of labor after the upper limits for duration is reached. This is 3 hours for nulliparous women and 2 hours for multiparous women.

Because of its size and rigidity, the fetal head has a major effect on the birth process. Which bones comprise the structure of the fetal skull? (Select all that apply.) a. Parietal b. Temporal c. Fontanel d. Occipital e. Femoral

ANS: A - Parietal B - Temporal D - Occipital The fetal skull has two parietal bones, two temporal bones, an occipital bone, and a frontal bone. The fontanels are membrane-filled spaces.

At least five factors affect the process of labor and birth. These are easily remembered as the five Ps. Which factors are included in this process? (Select all that apply.) a. Passenger b. Passageway c. Powers d. Pressure e. Psychologic response

ANS: A - Passenger B - Passageway C - Powers E - Psychologic response The five Ps are passenger (fetus and placenta), passageway (birth canal), powers (contractions), position of the mother, and psychologic response. Pressure is not one of the five Ps.

Most women with uncomplicated pregnancies can use the nurse as their primary source for nutritional information. However, the nurse or midwife may need to refer a client to a registered dietitian for in-depth nutritional counseling. Which conditions would require such a consultation? (Select all that apply.) a. Preexisting or gestational illness such as diabetes b. Ethnic or cultural food patterns c. Obesity d. Vegetarian diets e. Multifetal pregnancy

ANS: A - Preexisting or gestational illness such as diabetes B - Ethnic or cultural food patterns C - Obesity D - Vegetarian diets The nurse should be especially aware that conditions such as diabetes can require in-depth dietary planning and evaluation. To prevent issues with hypoglycemia and hyperglycemia, as well as an increased risk for perinatal morbidity and mortality, the client with a preexisting or gestational illness would benefit from a referral to a dietitian. Consultation with a dietitian may ensure that cultural food beliefs are congruent with modern knowledge of fetal development and that adjustments can be made to ensure that all nutritional needs are met. The obese pregnant client may be under the misapprehension that, because of her excess weight, little or no weight gain is necessary. According to the Institute of Medicine, a client with a BMI in the obese range should gain at least 7 kg to ensure a healthy outcome. This client may require in-depth counseling on the optimal food choices. The vegetarian client needs to have her dietary intake carefully assessed to ensure that the optimal combination of amino acids and protein intake is achieved. Very strict vegetarians (vegans) who consume only plant products may also require vitamin B and mineral supplementation. A multifetal pregnancy can be managed by increasing the number of servings of complex carbohydrates and proteins.

Women who have participated in childbirth education classes often bring a birth plan with them to the hospital. Which items might this plan include? (Select all that apply.) a. Presence of companions b. Clothing to be worn c. Care and handling of the newborn d. Medical interventions e. Date of delivery

ANS: A - Presence of companions B - Clothing to be worn C - Care and handling of the newborn D - Medical interventions The presence of companions, clothing to be worn, care and handling of the newborn, medical interventions, and environmental modifications all might be included in the couple's birth plan. Other items include the presence of nonessential medical personnel (students), labor activities such as the tub or ambulation, preferred comfort and relaxation methods, and any cultural or religious requirements. The expected date of delivery would not be part of a birth plan unless the client is scheduled for an elective cesarean birth.

Which factors influence cervical dilation? (Select all that apply.) a. Strong uterine contractions b. Force of the presenting fetal part against the cervix c. Size of the woman d. Pressure applied by the amniotic sac e. Scarring of the cervix

ANS: A - Strong uterine contractions B - Force of the presenting fetal part against the cervix D - Pressure applied by the amniotic sac E - Scarring of the cervix Dilation of the cervix occurs by the drawing upward of the musculofibrous components of the cervix, which is caused by strong uterine contractions. Pressure exerted by the amniotic fluid while the membranes are intact or by the force applied by the presenting part can also promote cervical dilation. Scarring of the cervix as a result of a previous infection or surgery may slow cervical dilation. Pelvic size or the size of the woman does not affect cervical dilation.

A woman has just moved to the United States from Mexico. She is 3 months pregnant and has arrived for her first prenatal visit. During her assessment interview, the nurse learns that the client has not had any immunizations. Which immunizations should she receive at this point in her pregnancy? (Select all that apply.) a. Tetanus b. Diphtheria c. Chickenpox d. Rubella e. Hepatitis B

ANS: A - Tetanus B - Diphtheria E - Hepatitis B Vaccines consisting of killed viruses may be used. Those that may be administered during pregnancy include tetanus, diphtheria, recombinant hepatitis B, and rabies vaccines. Immunizations with live or attenuated viruses are contraindicated during pregnancy because of their potential teratogenicity. Live-virus vaccines include those for measles (rubeola and rubella), chickenpox, and mumps

Which signs and symptoms should a woman immediately report to her health care provider? (Select all that apply.) a. Vaginal bleeding b. Rupture of membranes c. Heartburn accompanied by severe headache d. Decreased libido e. Urinary frequency

ANS: A - Vaginal bleeding B - Rupture of membranes C - Heartburn accompanied by severe headache Vaginal bleeding, rupture of membranes, and severe headaches are signs of potential complications in pregnancy. Clients should be advised to report these signs to their health care provider. Decreased libido and urinary frequency are common discomforts of pregnancy that do not require immediate health care interventions.

The class of drugs known as opioid analgesics (butorphanol, nalbuphine) is not suitable for administration to women with known opioid dependence. The antagonistic activity could precipitate withdrawal symptoms (abstinence syndrome) in both mothers and newborns. Which signs would indicate opioid or narcotic withdrawal in the mother? (Select all that apply.) a. Yawning, runny nose b. Increase in appetite c. Chills or hot flashes d. Constipation e. Irritability, restlessness

ANS: A - Yawning, runny nose C - Chills or hot flashes E - Irritability, restlessness The woman experiencing maternal opioid withdrawal syndrome will exhibit yawning, runny nose, sneezing, anorexia, chills or hot flashes, vomiting, diarrhea, abdominal pain, irritability, restlessness, muscle spasms, weakness, and drowsiness. Assessing both the mother and the newborn and planning the care accordingly are important steps for the nurse to take.

A tiered system of categorizing FHR has been recommended by professional organizations. Nurses, midwives, and physicians who care for women in labor must have a working knowledge of fetal monitoring standards and understand the significance of each category. What is the correct nomenclature for these categories? (Select all that apply.) a. Reassuring b. Category I c. Category II d. Nonreassuring e. Category III

ANS: B - Category I C - Category II E - Category III The three-tiered system of FHR tracings include category I, II, and III. Category I is a normal tracing requiring no action. Category II FHR tracings are indeterminate and includes tracings that do not meet category I or III criteria. Category III tracings are abnormal and require immediate intervention.

Foodborne illnesses can cause adverse effects for both mother and fetus. The nurse is in an ideal position to evaluate the client's knowledge regarding steps to prevent a foodborne illness. The nurse asks the client to "teach back" the fours simple steps of food preparation. What are they? (Select all that apply.) a. Purchase b. Clean c. Separate d. Cook e. Chill

ANS: B - Clean C - Separate D - Cook E - Chill According to the U.S. Food and Drug Administration (2013), the "four simple steps" are: • Clean: Frequently cleanse hands, food preparation surfaces, and utensils. • Separate: Avoid contact among raw meat, fish, or poultry and other foods that will not be cooked before consumption. • Cook: Cook foods to the proper temperature. • Chill: Properly store foods, and promptly refrigerate.

Which FHR decelerations would require the nurse to change the maternal position? (Select all that apply.) a. Early decelerations b. Late decelerations c. Variable decelerations d. Moderate decelerations e. Prolonged decelerations

ANS: B - Late decelerations C - Variable decelerations E - Prolonged decelerations Early decelerations (and accelerations) do not generally need any nursing intervention. Late decelerations suggest that the nurse should change the maternal position (lateral). Variable decelerations also require a maternal position change (side to side). Moderate decelerations are not an accepted category. Prolonged decelerations are late or variable decelerations that last for a prolonged period (longer than 2 minutes) and require intervention.

The number of routine laboratory tests during follow-up visits is limited; however, those that are performed are essential. Which statements regarding group B Streptococcus (GBS) testing are correct? (Select all that apply.) a. Performed between 32 and 34 weeks of gestation. b. Performed between 35 and 37 weeks of gestation. c. All women should be tested. d. Only women planning a vaginal birth should be tested. e. Women with a history of GBS should be retested.

ANS: B - Performed between 35 and 37 weeks of gestation. D - Only women planning a vaginal birth should be tested. E - Women with a history of GBS should be retested. GBS testing is recommended between 35 and 37 weeks of gestation; cultures collected earlier will not accurately predict the presence of GBS at birth. All women should be tested, even those planning an elective cesarean birth. Membranes may rupture early, requiring prophylactic antibiotics. Clients with a history of GBS should be retested.

Maternal hypotension is a potential side effect of regional anesthesia and analgesia. What nursing interventions could the nurse use to increase the client's blood pressure? (Select all that apply.) a. Place the woman in a supine position. b. Place the woman in a lateral position. c. Increase IV fluids. d. Administer oxygen. e. Perform a vaginal examination.

ANS: B - Place the woman in a lateral position. C - Increase IV fluids. D - Administer oxygen. Nursing interventions for maternal hypotension arising from analgesia or anesthesia include turning the woman to a lateral position, increasing IV fluids, administering oxygen via face mask, elevating the woman's legs, notifying the physician, administering an IV vasopressor, and monitoring the maternal and fetal status at least every 5 minutes until the woman is stable. Placing the client in a supine position causes venous compression, thereby limiting blood flow to and oxygenation of the placenta and fetus. A sterile vaginal examination has no bearing on maternal blood pressure.

According to the National Institute of Child Health and Human Development (NICHD) Three-Tier System of FHR Classification, category III tracings include all FHR tracings not categorized as category I or II. Which characteristics of the FHR belong in category III? (Select all that apply.) a. Baseline rate of 110 to 160 beats per minute b. Tachycardia c. Absent baseline variability not accompanied by recurrent decelerations d. Variable decelerations with other characteristics such as shoulders or overshoots e. Absent baseline variability with recurrent variable decelerations f. Bradycardia

ANS: B - Tachycardia D - Variable decelerations with other characteristics such as shoulders or overshoots E - Absent baseline variability with recurrent variable decelerations F - Bradycardia Tachycardia, variable decelerations with other characteristics, absent baseline variability with recurrent variable decelerations, and bradycardia are characteristics that are considered nonreassuring or abnormal and belong in category III. A FHR of 110 to 160 beats per minute is considered normal and belongs in category I. Absent baseline variability not accompanied by recurrent decelerations is a category II characteristic.

The baseline FHR is the average rate during a 10-minute segment. Changes in FHR are categorized as periodic or episodic. These patterns include both accelerations and decelerations. The labor nurse is evaluating the client's most recent 10-minute segment on the monitor strip and notes a late deceleration. Which is likely to have caused this change? (Select all that apply.) a. Spontaneous fetal movement b. Compression of the fetal head c. Placental abruption d. Cord around the baby's neck e. Maternal supine hypotension

ANS: C - Placental abruption E - Maternal supine hypotension Late decelerations are almost always caused by uteroplacental insufficiency. Insufficiency is caused by uterine tachysystole, maternal hypotension, epidural or spinal anesthesia, IUGR, intraamniotic infection, or placental abruption. Spontaneous fetal movement, vaginal examination, fetal scalp stimulation, fetal reaction to external sounds, uterine contractions, fundal pressure, and abdominal palpation are all likely to cause accelerations of the FHR. Early decelerations are most often the result of fetal head compression and may be caused by uterine contractions, fundal pressure, vaginal examination, and the placement of an internal electrode. A variable deceleration is likely caused by umbilical cord compression, which may happen when the umbilical cord is around the baby's neck, arm, leg, or other body part or when a short cord, a knot in the cord, or a prolapsed cord is present

During pregnancy, many changes occur as a direct result of the presence of the fetus. Which of these adaptations meet this criterion? (Select all that apply.) a. Leukorrhea b. Development of the operculum c. Quickening d. Ballottement e. Lightening

ANS: C - Quickening D - Ballottement E - Lightening Quickening is the first recognition of fetal movements or "feeling life." Quickening is often described as a flutter and is felt earlier in the multiparous woman than in the primiparous woman. Passive movement of the unengaged fetus is referred to as ballottement. Lightening occurs when the fetus begins to descend into the pelvis and occurs 2 weeks before labor in the nulliparous woman and at the start of labor in the multiparous woman. Leukorrhea is a white or slightly gray vaginal discharge that develops in response to cervical stimulation by estrogen and progesterone. Mucus fills the cervical canal creating a plug otherwise known as the operculum. The operculum acts as a barrier against bacterial invasion during the pregnancy

The BMI for a woman who is 51 kg before pregnancy and 1.57 m tall is _________.

ANS: 20.7 A commonly used method of evaluating the appropriateness of weight for height is the BMI, which is calculated by the following formula.

To provide optimal prenatal care, a blood pressure reading should be obtained at every prenatal visit. Calculating the mean arterial pressure (MAP) can increase the value of this diagnostic finding. MAP readings for a pregnant woman at term are 90+ = 5.8 mm Hg. The nurse has just obtained a BP of 106/70 mm Hg on a 37-week primiparous client. The formula for the MAP reading is (systolic + [2 diastolic]) 3. The MAP reading for this client is _____ mm Hg.

ANS: 82 The MAP can also be thought of as the mean of the blood pressure present in arterial circulation.

The nurse caring for a pregnant client is evaluating his or her health teaching regarding fetal circulation. Which statement from the client reassures the nurse that his or her teaching has been effective? a. "Optimal fetal circulation is achieved when I am in the side-lying position." b. "Optimal fetal circulation is achieved when I am on my back with a pillow under my knees." c. "Optimal fetal circulation is achieved when the head of the bed is elevated." d. "Optimal fetal circulation is achieved when I am on my abdomen."

ANS: A - "Optimal fetal circulation is achieved when I am in the side-lying position." Optimal circulation is achieved when the woman is lying at rest on her side. Decreased uterine circulation may lead to intrauterine growth restriction. Previously, it was believed that the left lateral position promoted maternal cardiac output, enhancing blood flow to the fetus. However, it is now known that the side-lying position enhances uteroplacental blood flow. If a woman lies on her back with the pressure of the uterus compressing the vena cava, then blood return to the right atrium is diminished. Although having the head of the bed elevated is recommended and ideal for later in pregnancy, the woman still must maintain a lateral tilt to the pelvis to avoid compressing the vena cava. Many women find lying on their abdomen uncomfortable as pregnancy advances. Side-lying is the ideal position to promote blood flow to the fetus.

After the nurse completes nutritional counseling for a pregnant woman, she asks the client to repeat the instructions to assess the client's understanding. Which statement indicates that the client understands the role of protein in her pregnancy? a. "Protein will help my baby grow." b. "Eating protein will prevent me from becoming anemic." c. "Eating protein will make my baby have strong teeth after he is born." d. "Eating protein will prevent me from being diabetic."

ANS: A - "Protein will help my baby grow." Protein is the nutritional element basic to growth. An adequate protein intake is essential to meeting the increasing demands of pregnancy. These demands arise from the rapid growth of the fetus; the enlargement of the uterus, mammary glands, and placenta; the increase in the maternal blood volume; and the formation of the amniotic fluid. Iron intake prevents anemia. Calcium intake is needed for fetal bone and tooth development. Glycemic control is needed in those with diabetes; protein is one nutritional factor to consider for glycemic control but not the primary role of protein intake.

A woman asks the nurse, "What protects my baby's umbilical cord from being squashed while the baby's inside of me?" What is the nurse's best response? a. "Your baby's umbilical cord is surrounded by connective tissue called Wharton's jelly, which prevents compression of the blood vessels." b. "Your baby's umbilical cord floats around in blood and amniotic fluid." c. "You don't need to be worrying about things like that." d. "The umbilical cord is a group of blood vessels that are very well protected by the placenta."

ANS: A - "Your baby's umbilical cord is surrounded by connective tissue called Wharton's jelly, which prevents compression of the blood vessels." Explaining the structure and function of the umbilical cord is the most appropriate response. Connective tissue called Wharton's jelly surrounds the umbilical cord, prevents compression of the blood vessels, and ensures continued nourishment of the embryo or fetus. The umbilical cord does not float around in blood or fluid. Telling the client not to worry negates her need for information and discounts her feelings. The placenta does not protect the umbilical cord.

Which nutritional recommendation regarding fluids is accurate? a. A woman's daily intake should be six to eight glasses of water, milk, and/or juice. b. Coffee should be limited to no more than 2 cups, but tea and cocoa can be consumed without worry. c. Of the artificial sweeteners, only aspartame has not been associated with any maternity health concerns. d. Water with fluoride is especially encouraged because it reduces the child's risk of tooth decay.

ANS: A - A woman's daily intake should be six to eight glasses of water, milk, and/or juice. Six to eight glasses is still the standard for fluids; however, they should be the right fluids. All beverages containing caffeine, including tea, cocoa, and some soft drinks, should be avoided or should be consumed only in limited amounts. Artificial sweeteners, including aspartame, have no ill effects on the normal mother or fetus. However, mothers with phenylketonuria (PKU) should avoid aspartame. Although no evidence indicates that prenatal fluoride consumption reduces childhood tooth decay, fluoride still helps the mother.

Which women should undergo prenatal testing for the human immunodeficiency virus (HIV)? a. All women, regardless of risk factors b. Women who have had more than one sexual partner c. Women who have had a sexually transmitted infection (STI) d. Woman who are monogamous with one partner

ANS: A - All women, regardless of risk factors An HIV test is recommended for all women, regardless of risk factors. The incidence of perinatal transmission from an HIV-positive mother to her fetus ranges from 25% to 35%. Women who test positive for HIV can then be treated.

What is the most likely cause for early decelerations in the fetal heart rate (FHR) pattern? a. Altered fetal cerebral blood flow b. Umbilical cord compression c. Uteroplacental insufficiency d. Spontaneous rupture of membranes

ANS: A - Altered fetal cerebral blood flow Early decelerations are the fetus' response to fetal head compression; these are considered benign, and interventions are not necessary. Variable decelerations are associated with umbilical cord compression. Late decelerations are associated with uteroplacental insufficiency. Spontaneous rupture of membranes has no bearing on the FHR unless the umbilical cord prolapses, which would result in variable or prolonged bradycardia.

Which presumptive sign or symptom of pregnancy would a client experience who is approximately 10 weeks of gestation? a. Amenorrhea b. Positive pregnancy test c. Chadwick sign d. Hegar sign

ANS: A - Amenorrhea Amenorrhea is a presumptive sign of pregnancy. Presumptive signs of pregnancy are those felt by the woman. A positive pregnancy test and the presence of the Chadwick and Hegar signs are all probable signs of pregnancy.

Which information regarding amniotic fluid is important for the nurse to understand? a. Amniotic fluid serves as a source of oral fluid and a repository for waste from the fetus. b. Volume of the amniotic fluid remains approximately the same throughout the term of a healthy pregnancy. c. The study of fetal cells in amniotic fluid yields little information. d. A volume of more than 2 L of amniotic fluid is associated with fetal renal abnormalities.

ANS: A - Amniotic fluid serves as a source of oral fluid and a repository for waste from the fetus. Amniotic fluid serves as a source of oral fluid, serves as a repository for waste from the fetus, cushions the fetus, and helps maintain a constant body temperature. The volume of amniotic fluid constantly changes. The study of amniotic fluid yields information regarding the sex of the fetus and the number of chromosomes. Too much amniotic fluid (hydramnios) is associated with gastrointestinal and other abnormalities.

A laboring woman is reclining in the supine position. What is the most appropriate nursing action at this time? a. Ask her to turn to one side. b. Elevate her feet and legs. c. Take her blood pressure. d. Determine whether fetal tachycardia is present.

ANS: A - Ask her to turn to one side The woman's supine position may cause the heavy uterus to compress her inferior vena cava, thus reducing blood return to her heart and reducing placental blood flow. Elevating her legs will not relieve the pressure from the inferior vena cava. If the woman is allowed to stay in the supine position and blood flow to the placental is reduced significantly, then fetal tachycardia may occur. The most appropriate nursing action is to prevent this from occurring by turning the woman to her side. Blood pressure readings may be obtained when the client is in the appropriate and safest position.

With regard to medications, herbs, boosters, and other substances normally encountered by pregnant women, what is important for the nurse to be aware of? a. Both prescription and over-the-counter (OTC) drugs that otherwise are harmless can be made hazardous by metabolic deficiencies of the fetus. b. The greatest danger of drug-caused developmental deficits in the fetus is observed in the final trimester. c. Killed-virus vaccines (e.g., tetanus) should not be administered during pregnancy, but live-virus vaccines (e.g., measles) are permissible. d. No convincing evidence exists that secondhand smoke is potentially dangerous to the fetus.

ANS: A - Both prescription and over-the-counter (OTC) drugs that otherwise are harmless can be made hazardous by metabolic deficiencies of the fetus. Both prescription and OTC drugs that otherwise are harmless can be made hazardous by metabolic deficiencies of the fetus. This is especially true for new medications and combinations of drugs. The greatest danger of drug-caused developmental defects exists in the interval from fertilization through the first trimester, during which a woman may not realize that she is pregnant. Live-virus vaccines should be part of postpartum care; killed-virus vaccines may be administered during pregnancy. Secondhand smoke is associated with fetal growth restriction and increases in infant mortality

A first-time mother at 18 weeks of gestation is in for her regularly scheduled prenatal visit. The client tells the nurse that she is afraid that she is going into premature labor because she is beginning to have regular contractions. The nurse explains that these are Braxton Hicks contractions. What other information is important for the nurse to share? a. Braxton Hicks contractions should be painless. b. They may increase in frequency with walking. c. These contractions might cause cervical dilation. d. Braxton Hicks contractions will impede oxygen flow to the fetus.

ANS: A - Braxton Hicks contractions should be painless. Soon after the fourth month of gestation, uterine contractions can be felt through the abdominal wall. Braxton Hicks contractions are regular and painless and continue throughout the pregnancy. Although they are not painful, some women complain that they are annoying. This type of contraction usually ceases with walking or exercise. Braxton Hicks contractions can be mistaken for true labor; however, they do not increase in intensity, frequency, or cause cervical dilation. These contractions facilitate uterine blood flow through the intervillous spaces of the placenta and thereby promote oxygen delivery to the fetus.

Which presentation is accurately described in terms of both the resenting part and the frequency of occurrence? a. Cephalic: occiput, at least 96% b. Breech: sacrum, 10% to 15% c. Shoulder: scapula, 10% to 15% d. Cephalic: cranial, 80% to 85%

ANS: A - Cephalic: occiput, at least 96% In cephalic presentations (head first), the presenting part is the occiput; this presentation occurs in 96% of births. In a breech birth, the sacrum emerges first; this presentation occurs in approximately 3% of births. In shoulder presentations, the scapula emerges first; this presentation occurs in only 1% of births. In a cephalic presentation, the part of the head or cranium that emerges first is the occiput; cephalic presentations occur in 96% of births

Which nursing intervention would result in an increase in maternal cardiac output? a. Change in position b. Oxytocin administration c. Regional anesthesia d. IV analgesic

ANS: A - Change in position Maternal supine hypotension syndrome is caused by the weight and pressure of the gravid uterus on the ascending vena cava when the woman is in a supine position. This position reduces venous return to the woman's heart, as well as cardiac output, and subsequently reduces her blood pressure. The nurse can encourage the woman to change positions and to avoid the supine position. Oxytocin administration, regional anesthesia, and IV analgesic may reduce maternal cardiac output.

While evaluating an external monitor tracing of a woman in active labor, the nurse notes that the FHR for five sequential contractions begins to decelerate late in the contraction, with the nadir of the decelerations occurring after the peak of the contraction. What is the nurse's first priority? a. Change the woman's position. b. Notify the health care provider. c. Assist with amnioinfusion d. Insert a scalp electrode.

ANS: A - Change the woman's position. Late FHR decelerations may be caused by maternal supine hypotension syndrome. These decelerations are usually corrected when the woman turns onto her side to displace the weight of the gravid uterus from the vena cava. If the fetus does not respond to primary nursing interventions for late decelerations, then the nurse should continue with subsequent intrauterine resuscitation measures and notify the health care provider. An amnioinfusion may be used to relieve pressure on an umbilical cord that has not prolapsed. The FHR pattern associated with this situation most likely will reveal variable decelerations. Although a fetal scalp electrode will provide accurate data for evaluating the well-being of the fetus, it is not a nursing intervention that will alleviate late decelerations nor is it the nurse's first priority.

Which information regarding the procedures and criteria for admitting a woman to the hospital labor unit is important for the nurse to understand? a. Client is considered to be in active labor when she arrives at the facility with contractions. b. Client can have only her male partner or predesignated doula with her at assessment. c. Children are not allowed on the labor unit. d. Non-English speaking client must bring someone to translate.

ANS: A - Client is considered to be in active labor when she arrives at the facility with contractions. According to the Emergency Medical Treatment and Active Labor Act (EMTALA), a woman is entitled to active labor care and is presumed to be in true labor until a qualified health care provider certifies otherwise. A woman may have anyone she wishes present for her support. An interpreter must be provided by the hospital, either in person or by a telephonic service. Siblings of the new infant may be allowed at the delivery, depending on hospital policy and adequate preparation and supervision.

Which statement best describes a normal uterine activity pattern in labor? a. Contractions every 2 to 5 minutes b. Contractions lasting approximately 2 minutes c. Contractions approximately 1 minute apart d. Contraction intensity of approximately 500 mm Hg with relaxation at 50 mm Hg

ANS: A - Contractions every 2 to 5 minutes Overall contraction frequency generally ranges from two to five contractions per 10 minutes of labor, with lower frequencies during the first stage and higher frequencies observed during the second stage. Contraction duration remains fairly stable throughout the first and second stages, ranging from 45 to 80 seconds, generally not exceeding 90 seconds. Contractions 1 minute apart are occurring too often and would be considered an abnormal labor pattern. The intensity of uterine contractions generally ranges from 25 to 50 mm Hg in the first stage of labor and may rise to more than 80 mm Hg in the second stage

What are the legal responsibilities of the perinatal nurses? a. Correctly interpreting FHR patterns, initiating appropriate nursing interventions, and documenting the outcomes b. Greeting the client on arrival, assessing her status, and starting an IV line c. Applying the external fetal monitor and notifying the health care provider d. Ensuring that the woman is comfortable

ANS: A - Correctly interpreting FHR patterns, initiating appropriate nursing interventions, and documenting the outcomes Nurses who care for women during childbirth are legally responsible for correctly interpreting FHR patterns, initiating appropriate nursing interventions based on those patterns, and documenting the outcomes of those interventions. Greeting the client on arrival, assessing her, and starting an IV line are activities that should be performed when any client arrives to the maternity unit. The nurse is not the only one legally responsible for performing these functions. Applying the external fetal monitor and notifying the health care provider is a nursing function that is part of the standard of care for all obstetric clients and falls within the registered nurse's scope of practice. Everyone caring for the pregnant woman should ensure that both she and her support partner are comfortable.

A client is experiencing back labor and complains of intense pain in her lower back. Which measure would best support this woman in labor? a. Counterpressure against the sacrum b. Pant-blow (breaths and puffs) breathing techniques c. Effleurage d. Conscious relaxation or guided imagery

ANS: A - Counterpressure against the sacrum Counterpressure is steady pressure applied by a support person to the sacral area with the fist or heel of the hand. This technique helps the woman cope with the sensations of internal pressure and pain in the lower back. The pain management techniques of pant-blow, effleurage, and conscious relaxation or guided imagery are usually helpful for contractions per the gate-control theory.

When assessing a woman in the first stage of labor, which clinical finding will alert the nurse that uterine contractions are effective? a. Dilation of the cervix b. Descent of the fetus to -2 station c. Rupture of the amniotic membranes d. Increase in bloody show

ANS: A - Dilation of the cervix The vaginal examination reveals whether the woman is in true labor. Cervical change, especially dilation, in the presence of adequate labor, indicates that the woman is in true labor. Engagement and descent of the fetus are not synonymous and may occur before labor. ROM may occur with or without the presence of labor. Bloody show may indicate a slow, progressive cervical change (e.g., effacement) in both true and false labor

Which condition is likely to be identified by the quadruple marker screen? a. Down syndrome b. Diaphragmatic hernia c. Congenital cardiac abnormality d. Anencephaly

ANS: A - Down syndrome The maternal serum level marker of alpha-fetoprotein is used to screen for Down syndrome, trisomy 18, neural tube defects, and other chromosomal anomalies. The quadruple-marker screen will not detect diaphragmatic hernia. Additional testing, such as ultrasonography, is required to diagnose diaphragmatic hernia. Congenital cardiac abnormality will most likely be identified during an ultrasound examination. The quadruple-marker screen will not detect anencephaly.

A pregnant woman reports that she is still playing tennis at 32 weeks of gestation. Which recommendation would the nurse make for this particular client after a tennis match? a. Drink several glasses of fluid. b. Eat extra protein sources such as peanut butter. c. Enjoy salty foods to replace lost sodium. d. Consume easily digested sources of carbohydrate.

ANS: A - Drink several glasses of fluid. If no medical or obstetric problems contraindicate physical activity, then pregnant women should get 30 minutes of moderate physical exercise daily. Liberal amounts of fluid should be consumed before, during, and after exercise because dehydration can trigger premature labor. The woman's caloric intake should be sufficient to meet the increased needs of pregnancy and the demands of exercise

Part of the nurse's role is assisting with pushing and positioning. Which guidance should the nurse provide to her client in active labor? a. Encourage the woman's cooperation in avoiding the supine position. b. Advise the woman to avoid the semi-Fowler position. c. Encourage the woman to hold her breath and tighten her abdominal muscles to produce a vaginal response. d. Instruct the woman to open her mouth and close her glottis, letting air escape after the push.

ANS: A - Encourage the woman's cooperation in avoiding the supine position. The woman should maintain a side-lying position. The semi-Fowler position is the recommended side-lying position with a lateral tilt to the uterus. Encouraging the woman to hold her breath and tighten her abdominal muscles is the Valsalva maneuver, which should be avoided. Both the mouth and glottis should be open, allowing air to escape during the push.

When managing the care of a woman in the second stage of labor, the nurse uses various measures to enhance the progress of fetal descent. Which instruction best describes these measures? a. Encouraging the woman to try various upright positions, including squatting and standing b. Telling the woman to start pushing as soon as her cervix is fully dilated c. Continuing an epidural anesthetic so pain is reduced and the woman can relax d. Coaching the woman to use sustained, 10- to 15-second, closed-glottis bearing-down efforts with each contraction

ANS: A - Encouraging the woman to try various upright positions, including squatting and standing Both upright and squatting positions may enhance the progress of fetal descent. Many factors dictate when a woman should begin pushing. Complete cervical dilation is necessary, but complete dilation is only one factor. If the fetal head is still in a higher pelvic station, then the physician or midwife may allow the woman to "labor down" if the woman is able (allowing more time for fetal descent and thereby reducing the amount of pushing needed). The epidural may mask the sensations and muscle control needed for the woman to push effectively. Closed glottic breathing may trigger the Valsalva maneuver, which increases intrathoracic and cardiovascular pressures, reducing cardiac output and inhibiting perfusion of the uterus and placenta. In addition, holding her breath for longer than 5 to 7 seconds diminishes the perfusion of oxygen across the placenta and results in fetal hypoxia.

Which nutrient's recommended dietary allowance (RDA) is higher during lactation than during pregnancy? a. Energy (kcal) b. Iron c. Vitamin A d. Folic acid

ANS: A - Energy (kcal) Nutrient needs for energy—protein, calcium, iodine, zinc, B vitamins, and vitamin C—remain higher during lactation than during pregnancy. The need for iron is not higher during lactation than during pregnancy. A lactating woman does not have a greater requirement for vitamin A than a nonpregnant woman. Folic acid requirements are the highest during the first trimester of pregnancy

A woman in labor passes some thick meconium as her amniotic fluid ruptures. The client asks the nurse where the baby makes the meconium. What is the correct response by the nurse? a. Fetal intestines b. Fetal kidneys c. Amniotic fluid d. Placenta

ANS: A - Fetal intestines As the fetus nears term, fetal waste products accumulate in the intestines as dark green-to-black, tarry meconium. Meconium is not produced by the fetal kidneys nor should it be present in the amniotic fluid, which may be an indication of fetal compromise. The placenta does not produce meconium

Which stage of labor varies the most in length? a. First b. Second c. Third d. Fourth

ANS: A - First The first stage of labor is considered to last from the onset of regular uterine contractions to the full dilation of the cervix. The first stage is significantly longer than the second and third stages combined. In a first-time pregnancy, the first stage of labor can take up to 20 hours. The second stage of labor lasts from the time the cervix is fully dilated to the birth of the fetus. The average length is 20 minutes for a multiparous woman and 50 minutes for a nulliparous woman. The third stage of labor lasts from the birth of the fetus until the placenta is delivered. This stage may be as short as 3 minutes or as long as 1 hour. The fourth stage of labor, recovery, lasts approximately 2 hours after the delivery of the placenta.

Which description of the four stages of labor is correct for both the definition and the duration? a. First stage: onset of regular uterine contractions to full dilation; less than 1 hour to 20 hours b. Second stage: full effacement to 4 to 5 cm; visible presenting part; 1 to 2 hours c. Third stage: active pushing to birth; 20 minutes (multiparous woman), 50 minutes (nulliparous woman) d. Fourth stage: delivery of the placenta to recovery; 30 minutes to 1 hour

ANS: A - First stage: onset of regular uterine contractions to full dilation; less than 1 hour to 20 hours Full dilation may occur in less than 1 hour, but in first-time pregnancies full dilation can take up to 20 hours. The second stage of labor extends from full dilation to birth and takes an average of 20 to 50 minutes, although 2 hours is still considered normal. The third stage of labor extends from birth to the expulsion of the placenta and usually takes a few minutes. The fourth stage begins after the expulsion of the placenta and lasts until homeostasis is reestablished (approximately 2 hours).

The nurse has formulated a diagnosis of Imbalanced nutrition: Less than body requirements for the client. Which goal is most appropriate for this client to obtain? a. Gain a total of 30 pounds. b. Consistently take daily supplements. c. Decrease her intake of snack foods. d. Increase her intake of complex carbohydrates.

ANS: A - Gain a total of 30 pounds. A weight gain of 30 pounds is one indication that the client has gained a sufficient amount for the nutritional needs of pregnancy. A daily supplement is not the best goal for this client and does not meet the basic need of proper nutrition during pregnancy. Decreasing snack foods may be needed and should be assessed; however, assessing weight gain is the best method of monitoring nutritional intake for this pregnant client. Although increasing the intake of complex carbohydrates is important for this client, monitoring the weight gain should be the end goal.

Which component of the physical examination are Leopold's maneuvers unable to determine? a. Gender of the fetus b. Number of fetuses c. Fetal lie and attitude d. Degree of the presenting part's descent into the pelvis

ANS: A - Gender of the fetus Leopold's maneuvers help identify the number of fetuses, the fetal lie and attitude, and the degree of descent of the presenting part into the pelvis. The gender of the fetus cannot be determined by performing Leopold's maneuvers.

Conscious relaxation is associated with which method of childbirth preparation? a. Grantly Dick-Read childbirth method b. Lamaze method c. Bradley method d. Psychoprophylactic method

ANS: A - Grantly Dick-Read childbirth method With the Grantly Dick-Read method, women are taught to consciously and progressively relax different muscle groups throughout the body until a high degree of skill at relaxation is achieved. The Lamaze method combines controlled muscular relaxation with breathing techniques. The Bradley method advocates natural labor, without any form of anesthesia or analgesia, assisted by a husband-coach and using breathing techniques for labor. The psychoprophylactic method is another name for the Lamaze method.

What is the primary role of the doula during labor? a. Helps the woman perform Lamaze breathing techniques and to provide support to the woman and her partner b. Checks the fetal monitor tracing for effects of the labor process on the fetal heart rate c. Takes the place of the father as a coach and support provider d. Administers pain medications as needed by the woman

ANS: A - Helps the woman perform Lamaze breathing techniques and to provide support to the woman and her partner A doula is professionally trained to provide labor support, including physical, emotional, and informational support, to both the woman and her partner during labor and the birth. The doula does not become involved with clinical tasks.

The nurse expects which maternal cardiovascular finding during labor? a. Increased cardiac output b. Decreased pulse rate c. Decreased white blood cell (WBC) count d. Decreased blood pressure

ANS: A - Increased cardiac output During each contraction, 400 ml of blood is emptied from the uterus into the maternal vascular system, which increases cardiac output by approximately 10% to 15% during the first stage of labor and by approximately 30% to 50% in the second stage of labor. The heart rate increases slightly during labor. The WBC count can increase during labor. During the first stage of labor, uterine contractions cause systolic readings to increase by approximately 10 mm Hg. During the second stage, contractions may cause systolic pressures to increase by 30 mm Hg and diastolic readings to increase by 25 mm Hg.

The nurse is caring for a client in early labor. Membranes ruptured approximately 2 hours earlier. This client is at increased risk for which complication? a. Intrauterine infection b. Hemorrhage c. Precipitous labor d. Supine hypotension

ANS: A - Intrauterine infection When the membranes rupture, microorganisms from the vagina can ascend into the amniotic sac, causing chorioamnionitis and placentitis. ROM is not associated with fetal or maternal bleeding. Although ROM may increase the intensity of the contractions and facilitate active labor, it does not result in precipitous labor. ROM has no correlation with supine hypotension

What is the most critical nursing action in caring for the newborn immediately after the birth? a. Keeping the airway clear b. Fostering parent-newborn attachment c. Drying the newborn and wrapping the infant in a blanket d. Administering eye drops and vitamin K

ANS: A - Keeping the airway clear The care given immediately after the birth focuses on assessing and stabilizing the newborn. Although fostering parent-newborn attachment is an important task for the nurse, it is not the most critical nursing action in caring for the newborn immediately after birth. The care given immediately after birth focuses on assessing and stabilizing the newborn. The nursing activities are (in order of importance) to maintain a patent airway, to support respiratory effort, and to prevent cold stress by drying the newborn and covering him or her with a warmed blanket or placing the newborn under a radiant warmer. After the newborn has been stabilized, the nurse assesses the newborn's physical condition, weighs and measures the newborn, administers prophylactic eye ointment and a vitamin K injection, affixes an identification bracelet, wraps the newborn in warm blankets, and then gives the newborn to the partner or to the mother of the infant.

Which information regarding protein in the diet of a pregnant woman is most helpful to the client? a. Many protein-rich foods are also good sources of calcium, iron, and B vitamins. b. Many women need to increase their protein intake during pregnancy. c. As with carbohydrates and fat, no specific recommendations exist for the amount of protein in the diet. d. High-protein supplements can be used without risk by women on macrobiotic diets.

ANS: A - Many protein-rich foods are also good sources of calcium, iron, and B vitamins. Good sources for protein, such as meat, milk, eggs, and cheese, have a lot of calcium and iron. Most women already eat a high-protein diet and do not need to increase their intake. Protein is sufficiently important that specific servings of meat and dairy are recommended. High-protein supplements are not recommended because they have been associated with an increased incidence of preterm births

Nursing care measures are commonly offered to women in labor. Which nursing measure reflects the application of the gate-control theory? a. Massage the woman's back. b. Change the woman's position. c. Give the prescribed medication. d. Encourage the woman to rest between contractions.

ANS: A - Massage the woman's back. According to the gate-control theory, pain sensations travel along sensory nerve pathways to the brain, but only a limited number of sensations, or messages, can travel through these nerve pathways at one time. Distraction techniques, such as massage or stroking, music, focal points, and imagery, reduce or completely block the capacity of the nerve pathways to transmit pain. These distractions are thought to work by closing down a hypothetic gate in the spinal cord, thus preventing pain signals from reaching the brain. The perception of pain is thereby diminished. Changing the woman's position, administering pain medication, and resting between contractions do not reduce or block the capacity of the nerve pathways to transmit pain using the gate-control theory

Which clinical finding or intervention might be considered the rationale for fetal tachycardia to occur? a. Maternal fever b. Umbilical cord prolapse c. Regional anesthesia d. Magnesium sulfate administration

ANS: A - Maternal fever Fetal tachycardia can be considered an early sign of fetal hypoxemia and may also result from maternal or fetal infection. Umbilical cord prolapse, regional anesthesia, and the administration of magnesium sulfate will each more likely result in fetal bradycardia, not tachycardia.

A pregnant client tells her nurse that she is worried about the blotchy, brownish coloring over her cheeks, nose, and forehead. The nurse can reassure her that this is a normal condition related to hormonal changes. What is the correct term for this integumentary finding? a. Melasma b. Linea nigra c. Striae gravidarum d. Palmar erythema

ANS: A - Melasma Melasma, (also called chloasma, the mask of pregnancy), usually fades after birth. This hyperpigmentation of the skin is more common in women with a dark complexion. Melasma appears in 50% to 70% of pregnant women. Linea nigra is a pigmented line that runs vertically up the abdomen. Striae gravidarum are also known as stretch marks. Palmar erythema is signified by pinkish red blotches on the hands.

As the United States and Canada continue to become more culturally diverse, recognizing a wide range of varying cultural beliefs and practices is increasingly important for the nursing staff. A client is from which country if she requests to have the baby's father in attendance? a. Mexico b. China c. Iran d. India

ANS: A - Mexico Hispanic women routinely have fathers and female relatives in attendance during the second stage of labor. The father of the baby is expected to provide encouragement, support, and reassurance that all will be well. In China, fathers are usually not present. The side-lying position is preferred for labor and birth because it is believed that this will reduce trauma to the infant. In China, the client has a stoic response to pain. In Iran, the father will not be present. Female support persons and female health care providers are preferred. For many, a male caregiver is unacceptable. In India, the father is usually not present, but female relatives are usually in attendance. Natural childbirth methods are preferred.

The nurse should be cognizant of which important information regarding nerve block analgesia and anesthesia? a. Most local agents are chemically related to cocaine and end in the suffix -caine. b. Local perineal infiltration anesthesia is effective when epinephrine is added, but it can be injected only once. c. Pudendal nerve block is designed to relieve the pain from uterine contractions. d. Pudendal nerve block, if performed correctly, does not significantly lessen the bearing-down reflex.

ANS: A - Most local agents are chemically related to cocaine and end in the suffix -caine. Common agents include lidocaine and chloroprocaine. Injections can be repeated to prolong the anesthesia. A pudendal nerve block relieves pain in the vagina, vulva, and perineum but not the pain from uterine contractions. A pudendal nerve block lessens or shuts down the bearing-down reflex

A client has arrived for her first prenatal appointment. She asked the nurse to explain exactly how long the pregnancy will be. What is the nurse's best response? a. Normal pregnancy is 10 lunar months. b. Pregnancy is made up of four trimesters. c. Pregnancy is considered term at 36 weeks. d. Estimated date of delivery (EDD) is 40 completed weeks.

ANS: A - Normal pregnancy is 10 lunar months. Pregnancy spans 9 calendar months; but, health care providers prefer to use the lunar month of 28 days or 4 weeks. Pregnancy consists of three trimesters, each approximately 13 weeks long. A pregnancy is considered term at 37 completed weeks; however, EDD is based upon 40 weeks of gestation.

Under which circumstance should the nurse assist the laboring woman into a hands-and-knees position? a. Occiput of the fetus is in a posterior position. b. Fetus is at or above the ischial spines. c. Fetus is in a vertex presentation. d. Membranes have ruptured.

ANS: A - Occiput of the fetus is in a posterior position. The hands-and-knees position is effective in helping to rotate the fetus from a posterior to an anterior position. Many women experience the irresistible urge to push when the fetus is at the level of the ischial spines. In some cases, this urge is felt before the woman is fully dilated. The woman should be instructed not to push until complete cervical dilation has occurred. No one position is correct for childbirth. The two most common positions assumed by women are the sitting and side-lying positions. The woman may be encouraged into a hands-and-knees position if the umbilical cord prolapsed when the membranes ruptured

What is the correct term used to describe the mucous plug that forms in the endocervical canal? a. Operculum b. Leukorrhea c. Funic souffle d. Ballottement

ANS: A - Operculum The operculum protects against bacterial invasion. Leukorrhea is the mucus that forms the endocervical plug (the operculum). The funic souffle is the sound of blood flowing through the umbilical vessels. Ballottement is a technique for palpating the fetus.

What is the correct placement of the tocotransducer for effective EFM? a. Over the uterine fundus b. On the fetal scalp c. Inside the uterus d. Over the mother's lower abdomen

ANS: A - Over the uterine fundus The tocotransducer monitors uterine activity and should be placed over the fundus, where the most intensive uterine contractions occur. The tocotransducer is for external use.

A nurse caring for a woman in labor should understand that absent or minimal variability is classified as either abnormal or indeterminate. Which condition related to decreased variability is considered benign? a. Periodic fetal sleep state b. Extreme prematurity c. Fetal hypoxemia d. Preexisting neurologic injury

ANS: A - Periodic fetal sleep state When the fetus is temporarily in a sleep state, minimal variability is present. Periodic fetal sleep states usually last no longer than 30 minutes. A woman in labor with extreme prematurity may display a FHR pattern of minimal or absent variability. Abnormal variability may also be related to fetal hypoxemia and metabolic acidemia. Congenital anomalies or a preexisting neurologic injury may also result in absent or minimal variability. Other possible causes might be central nervous system (CNS) depressant medications, narcotics, or general anesthesia

The nurse should be cognizant of which physiologic effect of pain? a. Predominant pain of the first stage of labor is visceral pain that is located in the lower portion of the abdomen. b. Referred pain is the extreme discomfort experienced between contractions. c. Somatic pain of the second stage of labor is more generalized and related to fatigue. d. Pain during the third stage is a somewhat milder version of the pain experienced during the second stage.

ANS: A - Predominant pain of the first stage of labor is visceral pain that is located in the lower portion of the abdomen. Predominant pain comes from cervical changes, the distention of the lower uterine segment, and uterine ischemia. Referred pain occurs when the pain that originates in the uterus radiates to the abdominal wall, lumbosacral area of the back, iliac crests, and gluteal area. Second-stage labor pain is intense, sharp, burning, and localized. Third-stage labor pain is similar to that of the first stage.

What is the nurse's understanding of the appropriate role of primary and secondary powers? a. Primary powers are responsible for the effacement and dilation of the cervix. b. Effacement is generally well ahead of dilation in women giving birth for the first time; they are closer together in subsequent pregnancies. c. Scarring of the cervix caused by a previous infection or surgery may make the delivery a bit more painful, but it should not slow or inhibit dilation. d. Pushing in the second stage of labor is more effective if the woman can breathe deeply and control some of her involuntary needs to push, as the nurse directs.

ANS: A - Primary powers are responsible for the effacement and dilation of the cervix. The primary powers are responsible for dilation and effacement; secondary powers are concerned with expulsion of the fetus. Effacement is generally well ahead of dilation in first-time pregnancies; they are closer together in subsequent pregnancies. Scarring of the cervix may slow dilation. Pushing is more effective and less fatiguing when the woman begins to push only after she has the urge to do so.

What is the correct term for a woman who has completed one pregnancy with a fetus (or fetuses) reaching the stage of fetal viability? a. Primipara b. Primigravida c. Multipara d. Nulligravida

ANS: A - Primipara A primipara is a woman who has completed one pregnancy with a viable fetus. To help remember the terms: gravida is a pregnant woman; para comes from parity, meaning a viable fetus; primi means first; multi means many; and null means none. Therefore, a primigravida is a woman pregnant for the first time; a multipara is a woman who has completed two or more pregnancies with a viable fetus; and a nulligravida is a woman who has never been pregnant.

In recovery, if a woman is asked to either raise her legs (knees extended) off the bed or flex her knees, and then place her feet flat on the bed and raise her buttocks well off the bed, the purpose of this exercise is to assess what? a. Recovery from epidural or spinal anesthesia b. Hidden bleeding underneath her c. Flexibility d. Whether the woman is a candidate to go home after 6 hours

ANS: A - Recovery from epidural or spinal anesthesia If the numb or prickly sensations are gone from her legs after these movements, then she has likely recovered from the epidural or spinal anesthesia. Assessing the client for bleeding beneath her buttocks before discharge from the recovery is always important; however, she should be rolled to her side for this assessment. The nurse is not required to assess the woman for flexibility. This assessment is performed to evaluate whether the client has recovered from spinal anesthesia, not to determine if she is a candidate for early discharge

What kind of fetal anomalies are most often associated with oligohydramnios? a. Renal b. Cardiac c. Gastrointestinal d. Neurologic

ANS: A - Renal An amniotic fluid volume of less than 300 ml (oligohydramnios) is often associated with fetal renal anomalies. The amniotic fluid volume has no bearing on the fetal cardiovascular system. Gastrointestinal anomalies are associated with hydramnios or an amniotic fluid volume greater than 2 L. The amniotic fluid volume has no bearing on the fetal neurologic system.

The client has delivered by urgent caesarean birth for fetal compromise. Umbilical cord gases were obtained for acid-base determination. The pH is 6.9, partial pressure of carbon dioxide (PCO2) is elevated, and the base deficit is 11 mmol/L. What type of acidemia is displayed by the infant? a. Respiratory b. Metabolic c. Mixed d. Turbulent

ANS: A - Respiratory These findings are evidence of respiratory acidemia. Metabolic acidemia is expressed by a pH <7.20, normal carbon dioxide pressure, and a base excess of 12 mmol/L. Mixed acidemia is evidenced by a pH <7.20, elevated carbon dioxide pressure, and a base excess of 12 mmol/L. There is no such finding as turbulent acidemia.

Which technique is an adequate means of controlling the birth of the fetal head during delivery in a vertex presentation? a. Ritgen maneuver b. Fundal pressure c. Lithotomy position d. De Lee apparatus

ANS: A - Ritgen maneuver The Ritgen maneuver extends the head during the actual birth and protects the perineum. Gentle, steady pressure against the fundus of the uterus facilitates vaginal birth. The lithotomy position has been commonly used in Western cultures, partly because it is convenient for the health care provider. The De Lee apparatus is used to suction fluid from the infant's mouth.

Which alterations in the perception of pain by a laboring client should the nurse understand? a. Sensory pain for nulliparous women is often greater than for multiparous women during early labor. b. Affective pain for nulliparous women is usually less than for multiparous women throughout the first stage of labor. c. Women with a history of substance abuse experience more pain during labor. d. Multiparous women have more fatigue from labor and therefore experience more pain.

ANS: A - Sensory pain for nulliparous women is often greater than for multiparous women during early labor. Sensory pain is greater for nulliparous women because their reproductive tract structures are less supple. Affective pain is greater for nulliparous women during the first stage but decreases for both nulliparous and multiparous during the second stage. Women with a history of substance abuse experience the same amount of pain as those without such a history. Nulliparous women have longer labors and therefore experience more fatigue.

Which behavior indicates that a woman is "seeking safe passage" for herself and her infant? a. She keeps all prenatal appointments. b. She "eats for two." c. She slowly drives her car. d. She wears only low-heeled shoes.

ANS: A - She keeps all prenatal appointments The goal of prenatal care is to foster a safe birth for the infant and mother. Although properly eating, carefully driving, and using proper body mechanics all are healthy measures that a mother can take, obtaining prenatal care is the optimal method for providing safety for both herself and her baby

Breathing patterns are taught to laboring women. Which breathing pattern should the nurse support for the woman and her coach during the latent phase of the first stage of labor if the couple has attended childbirth preparation classes? a. Slow-paced breathing b. Deep abdominal breathing c. Modified-paced breathing d. Patterned-paced breathing

ANS: A - Slow-paced breathing Slow-paced breathing is approximately one half the woman's normal breathing rate and is used during the early stages of labor when a woman can no longer walk or talk through her contractions. No such pattern called deep abdominal breathing exists in childbirth preparation. Modified-paced breathing is shallow breathing that is twice the woman's normal breathing rate. It is used when labor progresses and the woman can no longer maintain relaxation through paced breathing. Patterned-pace breathing is a fast, 4:1 breathe, breathe, breathe, blow pattern that is used during the transitional phase of labor just before pushing and delivery

Why might it be more difficult to diagnose appendicitis during pregnancy? a. The appendix is displaced upward and laterally, high and to the right. b. The appendix is displaced upward and laterally, high and to the left. c. The appendix is deep at the McBurney's point. d. The appendix is displaced downward and laterally, low and to the right.

ANS: A - The appendix is displaced upward and laterally, high and to the right. The appendix is displaced high and to the right, not to the left. It is displaced beyond the McBurney's point and is not displaced in a downward direction.

What is important for the nurse to recognize regarding the new father and his acceptance of the pregnancy and preparation for childbirth? a. The father goes through three phases of acceptance of his own. b. The father's attachment to the fetus cannot be as strong as that of the mother because it does not start until after the birth. c. In the last 2 months of pregnancy, most expectant fathers suddenly get very protective of their established lifestyle and resist making changes to the home. d. Typically, men remain ambivalent about fatherhood right up to the birth of their child.

ANS: A - The father goes through three phases of acceptance of his own. A father typically goes through three phases of development to reach acceptance of fatherhood: the announcement phase, the moratorium phase, and the focusing phase. The father-child attachment can be as strong as the mother-child relationship and can also begin during pregnancy. During the last 2 months of the pregnancy, many expectant fathers work hard to improve the environment of the home for the child. Typically, the expectant father's ambivalence ends by the first trimester, and he progresses to adjusting to the reality of the situation and then to focusing on his role

When assessing a multiparous woman who has just given birth to an 8-pound boy, the nurse notes that the woman's fundus is firm and has become globular in shape. A gush of dark red blood comes from her vagina. What is the nurse's assessment of the situation? a. The placenta has separated. b. A cervical tear occurred during the birth. c. The woman is beginning to hemorrhage. d. Clots have formed in the upper uterine segment.

ANS: A - The placenta has separated. Placental separation is indicated by a firmly contracting uterus, a change in the uterus from a discoid to a globular ovoid shape, a sudden gush of dark red blood from the introitus, an apparent lengthening of the umbilical cord, and a finding of vaginal fullness. Cervical tears that do not extend to the vagina result in minimal blood loss. Signs of hemorrhage are a boggy uterus, bright red vaginal bleeding, alterations in vital signs, pallor, lightheadedness, restlessness, decreased urinary output, and alteration in the level of consciousness. If clots have formed in the upper uterine segment, then the nurse would expect to find the uterus boggy and displaced to the side.

A woman is in her seventh month of pregnancy. She has been complaining of nasal congestion and occasional epistaxis. Which statement best describes why this may be happening to this client? a. This respiratory change is normal in pregnancy and caused by an elevated level of estrogen. b. This cardiovascular change is abnormal, and the nosebleeds are an ominous sign. c. The woman is a victim of domestic violence and is being hit in the face by her partner. d. The woman has been intranasally using cocaine.

ANS: A - This respiratory change is normal in pregnancy and caused by an elevated level of estrogen. Elevated levels of estrogen cause capillaries to become engorged in the respiratory tract, which may result in edema in the nose, larynx, trachea, and bronchi. This congestion may cause nasal stuffiness and epistaxis. Cardiovascular changes in pregnancy may cause edema in the lower extremities. Domestic violence cannot be determined on the basis on the sparse facts provided. If the woman had been hit in the face, then she most likely would have additional physical findings. Cocaine use cannot be determined on the basis on the sparse facts provided.

The nurse should be aware of which information related to a woman's intake and output during labor? a. Traditionally, restricting the laboring woman to clear liquids and ice chips is being challenged because regional anesthesia is used more often than general anesthesia. b. Intravenous (IV) fluids are usually necessary to ensure that the laboring woman stays hydrated. c. Routine use of an enema empties the rectum and is very helpful for producing a clean, clear delivery. d. When a nulliparous woman experiences the urge to defecate, it often means birth will quickly follow.

ANS: A - Traditionally, restricting the laboring woman to clear liquids and ice chips is being challenged because regional anesthesia is used more often than general anesthesia. Women are awake with regional anesthesia and are able to protect their own airway, which reduces the worry over aspiration. Routine IV fluids during labor are unlikely to be beneficial and may be harmful. The routine use of an enema is, at best, ineffective and may be harmful. Having the urge to defecate followed by the birth of her fetus is true for a multiparous woman but not for a nulliparous woman

Which guidance might the nurse provide for a client with severe morning sickness? a. Trying lemonade and potato chips b. Drinking plenty of fluids early in the day c. Immediately brushing her teeth after eating d. Never snacking before bedtime

ANS: A - Trying lemonade and potato chips Interestingly, some women can tolerate tart or salty foods when they are nauseated. Lemonade and potato chips are an ideal combination. The woman should avoid drinking too much when nausea is most likely, but she should increase her fluid levels later in the day when she feels better. The woman should avoid brushing her teeth immediately after eating. A small snack of cereal and milk or yogurt before bedtime may help the stomach in the morning

A client at 34 weeks of gestation seeks guidance from the nurse regarding personal hygiene. Which information should the nurse provide? a. Tub bathing is permitted even in late pregnancy unless membranes have ruptured. b. The perineum should be wiped from back to front. c. Bubble bath and bath oils are permissible because they add an extra soothing and cleansing action to the bath. d. Expectant mothers should use specially treated soap to cleanse the nipples.

ANS: A - Tub bathing is permitted even in late pregnancy unless membranes have ruptured. The primary danger from taking baths is falling in the tub. The perineum should be wiped from front to back. Bubble baths and bath oils should be avoided because they may irritate the urethra. Soap, alcohol, ointments, and tinctures should not be used to cleanse the nipples because they remove protective oils. Warm water is sufficient.

An expectant couple attending childbirth classes have questions regarding multiple births since twins "run in the family." What information regarding multiple births is important for the nurse to share? a. Twinning and other multiple births are increasing because of the use of fertility drugs and delayed childbearing. b. Dizygotic twins (two fertilized ova) have the potential to be conjoined twins. c. Identical twins are more common in Caucasian families. d. Fraternal twins are the same gender, usually male.

ANS: A - Twinning and other multiple births are increasing because of the use of fertility If the parents-to-be are older and have taken fertility drugs, then they would be very interested to know about twinning and other multiple births. Conjoined twins are monozygotic; that is, they are from a single fertilized ovum in which division occurred very late. Identical twins show no racial or ethnic preference, and fraternal twins are more common among African-American women. Fraternal twins can be different genders or the same gender, and identical twins are the same gender.

Which alteration in the FHR pattern would indicate the potential need for an amnioinfusion? a. Variable decelerations b. Late decelerations c. Fetal bradycardia d. Fetal tachycardia

ANS: A - Variable decelerations Amnioinfusion is used during labor to either dilute meconium-stained amniotic fluid or supplement the amount of amniotic fluid to reduce the severity of variable FHR decelerations caused by cord compression. Late decelerations are unresponsive to amnioinfusion. Amnioinfusion is not appropriate for the treatment of fetal bradycardia and has no bearing on fetal tachycardia.

A new mother asks the nurse about the "white substance" covering her infant. How should the nurse explain the purpose of vernix caseosa? a. Vernix caseosa protects the fetal skin from the amniotic fluid. b. Vernix caseosa promotes the normal development of the peripheral nervous system. c. Vernix caseosa allows the transport of oxygen and nutrients across the amnion. d. Vernix caseosa regulates fetal temperature.

ANS: A - Vernix caseosa protects the fetal skin from the amniotic fluid. Prolonged exposure to the amniotic fluid during the fetal period could result in the breakdown of the skin without the protection of the vernix caseosa. Normal development of the peripheral nervous system was dependent on nutritional intake of the mother. The amnion was the inner membrane that surrounded the fetus and was not involved in the oxygen and nutrient exchange. The amniotic fluid helped maintain fetal temperature.

Which development related to the integumentary system is correct? a. Very fine hairs called lanugo appear at 12 weeks of gestation. b. Eyelashes, eyebrows, and scalp hair appear at 28 weeks of gestation. c. Fingernails and toenails develop at 28 weeks of gestation. d. By 32 weeks, scalp hair becomes apparent.

ANS: A - Very fine hairs called lanugo appear at 12 weeks of gestation. Very fine hairs, called lanugo appear first at 12 weeks of gestational age on the fetus' eyebrows and upper lip. By 20 weeks of gestation, lanugo covers the entire body. By 20 weeks of gestation the eyelashes, eyebrows, and scalp hair also begin to grow. By 28 weeks of gestation, the scalp hair is longer than these fine hairs, which is thin and may disappear by term. Fingernails and toenails develop from thickened epidermis, beginning during the 10th week. Fingernails reach the fingertips at 32 weeks of gestation, and the toenails reach the toe tips at 36 weeks of gestation.

The various systems and organs of the fetus develop at different stages. Which statement is most accurate? a. Cardiovascular system is the first organ system to function in the developing human. b. Hematopoiesis originating in the yolk sac begins in the liver at 10 weeks of gestation. c. Body changes from straight to C-shape occurs at 8 weeks of gestation. d. Gastrointestinal system is mature at 32 weeks of gestation.

ANS: A -Cardiovascular system is the first organ system to function in the developing human. The heart is developmentally complete by the end of the embryonic stage. Hematopoiesis begins in the liver during the sixth week. The body becomes C-shaped at 21 weeks of gestation. The gastrointestinal system is complete at 36 weeks of gestation

Which statement made by a lactating woman leads the nurse to believe that the client might have lactose intolerance? a. "I always have heartburn after I drink milk." b. "If I drink more than a cup of milk, I usually have abdominal cramps and bloating." c. "Drinking milk usually makes me break out in hives." d. "Sometimes I notice that I have bad breath after I drink a cup of milk."

ANS: B - "If I drink more than a cup of milk, I usually have abdominal cramps and bloating." Lactose intolerance, which is an inability to digest milk sugar because of a lack of the enzyme lactose in the small intestine, is a problem that interferes with milk consumption. Milk consumption may cause abdominal cramping, bloating, and diarrhea in such people, although many lactose-intolerant individuals can tolerate small amounts of milk without symptoms. A woman with lactose intolerance is more likely to experience bloating and cramping, not heartburn. A client who breaks out in hives after consuming milk is more likely to have a milk allergy and should be advised to simply brush her teeth after consuming dairy products.

A woman who is 39 weeks pregnant expresses fear about her impending labor and how she will manage. What is the nurse's ideal response? a. "Don't worry about it. You'll do fine." b. "It's normal to be anxious about labor. Let's discuss what makes you afraid." c. "Labor is scary to think about, but the actual experience isn't." d. "You can have an epidural. You won't feel anything."

ANS: B - "It's normal to be anxious about labor. Let's discuss what makes you afraid." "It's normal to be anxious about labor. Let's discuss what makes you afraid" is a statement that allows the woman to share her concerns with the nurse and is a therapeutic communication tool. "Don't worry about it. You'll do fine" negates the woman's fears and is not therapeutic. "Labor is scary to think about, but the actual experience isn't" negates the woman's fears and offers a false sense of security. To suggest that every woman can have an epidural is untrue. A number of criteria must be met before an epidural is considered. Furthermore, many women still experience the feeling of pressure with an epidural

A patient in her first trimester complains of nausea and vomiting. She asks, "Why does this happen?" What is the nurse's best response? a. "Nausea and vomiting are due to an increase in gastric motility." b. "Nausea and vomiting may be due to changes in hormones." c. "Nausea and vomiting are related to an increase in glucose levels." d. "Nausea and vomiting are caused by a decrease in gastric secretions."

ANS: B - "Nausea and vomiting may be due to changes in hormones." Nausea and vomiting are believed to be caused by increased levels of hormones, decreased gastric motility, and hypoglycemia. Gastric motility decreases during pregnancy. Glucose levels decrease in the first trimester. Although gastric secretions decrease, these secretions are not the primary cause of the nausea and vomiting.

A first-time mother is concerned about the type of medications she will receive during labor. The client is in a fair amount of pain and is nauseated. In addition, she appears to be very anxious. The nurse explains that opioid analgesics are often used along with sedatives. How should the nurse phrase the rationale for this medication combination? a. "The two medications, together, reduce complications." b. "Sedatives enhance the effect of the pain medication." c. "The two medications work better together, enabling you to sleep until you have the baby." d. "This is what your physician has ordered for you."

ANS: B - "Sedatives enhance the effect of the pain medication." Sedatives may be used to reduce the nausea and vomiting that often accompany opioid use. In addition, some ataractic drugs reduce anxiety and apprehension and potentiate the opioid analgesic affects. A potentiator may cause two drugs to work together more effectively, but it does not ensure zero maternal or fetal complications. Sedation may be a related effect of some ataractic drugs; however, sedation is not the goal. Furthermore, a woman is unlikely to be able to sleep through transitional labor and birth. Although the physician may have ordered the medication, "This is what your physician has ordered for you" is not an acceptable comment for the nurse to make.

A woman who is 16 weeks pregnant asks the nurse, "Is it possible to tell by ultrasound if the baby is a boy or girl yet?" What is the best answer? a. "A baby's sex is determined as soon as conception occurs." b. "The baby has developed enough to enable us to determine the sex by examining the genitals through an ultrasound scan." c. "Boys and girls look alike until approximately 20 weeks after conception, and then they begin to look different." d. "It might be possible to determine your baby's sex, but the external organs look very similar right now."

ANS: B - "The baby has developed enough to enable us to determine the sex by examining the genitals through an ultrasound scan." Although gender is determined at conception, the external genitalia of males and females look similar through the ninth week. By the twelfth week, the external genitalia are distinguishable as male or female.

A new client and her partner arrive on the labor, delivery, recovery, and postpartum (LDRP) unit for the birth of their first child. The nurse applies the electronic fetal monitor (EFM) to the woman. Her partner asks you to explain what is printing on the graph, referring to the EFM strip. He wants to know what the baby's heart rate should be. What is the nurse's best response? a. "Don't worry about that machine; that's my job." b. "The baby's heart rate will fluctuate in response to what is happening during labor." c. "The top line graphs the baby's heart rate, and the bottom line lets me know how strong the contractions are." d. "Your physician will explain all of that later."

ANS: B - "The baby's heart rate will fluctuate in response to what is happening during labor." Explaining what indicates a normal FHR teaches the partner about fetal monitoring and provides support and information to alleviate his fears. Telling the partner not to worry discredits his feelings and does not provide the teaching he is requesting. Telling the partner that the graph indicates how strong the contractions are provides inaccurate information and does not address the partner's concerns about the FHR. The EFM graphs the frequency and duration of the contractions, not their intensity. Nurses should take every opportunity to provide teaching to the client and her family, especially when information is requested

A woman is 15 weeks pregnant with her first baby. She asks how long it will be before she feels the baby move. What is the nurse's best answer? a. "You should have felt the baby move by now." b. "Within the next month, you should start to feel fluttering sensations." c. "The baby is moving; however, you can't feel it yet." d. "Some babies are quiet, and you don't feel them move."

ANS: B - "Within the next month, you should start to feel fluttering sensations." Maternal perception of fetal movement usually begins 16 to 20 weeks after conception. Because this is her first pregnancy, movement is felt toward the later part of the 16- to 20-week time period. Stating, "you should have felt the baby move by now" is incorrect and may be an alarming statement to the client. Fetal movement should be felt by 16 to 20 weeks. If movement is not felt by the end of that time, then further assessment is necessary.

A newly married couple plans to use the natural family planning method of contraception. Understanding how long an ovum can live after ovulation is important to them. The nurse knows that his or her teaching was effective when the couple responds that an ovum is considered fertile for which period of time? a. 6 to 8 hours b. 24 hours c. 2 to 3 days d. 1 week

ANS: B - 24 hours Most ova remain fertile for approximately 24 hours after ovulation, much longer than 6 to 8 hours. However, ova do not remain fertile for 2 to 3 days or are viable for 1 week. If unfertilized by a sperm after 24 hours, the ovum degenerates and is reabsorbed.

A woman's obstetric history indicates that she is pregnant for the fourth time, and all of her children from previous pregnancies are living. One was born at 39 weeks of gestation, twins were born at 34 weeks of gestation, and another child was born at 35 weeks of gestation. What is her gravidity and parity using the GTPAL system? a. 3-1-1-1-3 b. 4-1-2-0-4 c. 3-0-3-0-3 d. 4-2-1-0-3

ANS: B - 4-1-2-0-4 Using the GTPAL system, 4-1-2-0-4 is the correct calculation of this woman's gravidity and parity. The numbers reflect the woman's gravidity and parity information. Her information is calculated as: G reflects the total number of times the woman has been pregnant; she is pregnant for the fourth time. T indicates the number of pregnancies carried to term, not the number of deliveries at term; only one of her pregnancies resulted in a fetus at term. P is the number of pregnancies that resulted in a preterm birth; the woman has had two pregnancies in which she delivered preterm. A signifies whether the woman has had any abortions or miscarriages before the period of viability; she has not. L signifies the number of children born who are currently living; the woman has four children. 3-1-1-1-3 is an incorrect calculation of this woman's gravidity and parity; 3-0-3-0-3 is an incorrect calculation of this woman's gravidity and parity; and 4-2-1-0-3 is an incorrect calculation of this woman's gravidity and parity.

What is the role of the nurse as it applies to informed consent? a. Inform the client about the procedure, and ask her to sign the consent form. b. Act as a client advocate, and help clarify the procedure and the options. c. Call the physician to see the client. d. Witness the signing of the consent form.

ANS: B - Act as a client advocate, and help clarify the procedure and the options. Nurses play a part in the informed consent process by clarifying and describing procedures or by acting as the woman's advocate and asking the primary health care provider for further explanations. The physician is responsible for informing the woman of her options, explaining the procedure, and advising the client about potential risk factors. The physician must be present to explain the procedure to the client. However, the nurse's responsibilities go further than simply asking the physician to see the client. The nurse may witness the signing of the consent form. However, depending on the state's guidelines, the woman's husband or another hospital health care employee may sign as a witness.

Nurses can help their clients by keeping them informed about the distinctive stages of labor. Which description of the phases of the first stage of labor is accurate? a. Latent: Mild, regular contractions; no dilation; bloody show; duration of 2 to 4 hours b. Active: Moderate, regular contractions; 4- to 7-cm dilation; duration of 3 to 6 hours c. Lull: No contractions; dilation stable; duration of 20 to 60 minutes d. Transition: Very strong but irregular contractions; 8- to 10-cm dilation; duration of 1 to 2 hours

ANS: B - Active: Moderate, regular contractions; 4- to 7-cm dilation; duration of 3 to 6 hours The active phase is characterized by moderate, regular contractions; 4- to 7-cm dilation; and a duration of 3 to 6 hours. The latent phase is characterized by mild-to-moderate and irregular contractions; dilation up to 3 cm; brownish-to-pale pink mucus, and a duration of 6 to 8 hours. No official "lull" phase exists in the first stage. The transition phase is characterized by strong- to-very strong and regular contractions; 8- to 10-cm dilation; and a duration of 20 to 40 minutes.

Which description of the phases of the first stage of labor is most accurate? a. Latent: mild, regular contractions; no dilation; bloody show b. Active: moderate, regular contractions; 4 to 7 cm dilation c. Lull: no contractions; dilation stable d. Transition: very strong but irregular contractions; 8 to 10 cm dilation \

ANS: B - Active: moderate, regular contractions; 4 to 7 cm dilation The active phase is characterized by moderate and regular contractions, 4 to 7 cm dilation, and duration of 3 to 6 hours. The latent phase is characterized by mild-to-moderate and irregular contractions, dilation up to 3 cm, brownish-to-pale pink mucus, and duration of 6 to 8 hours. No official "lull" phase exists in the first stage. The transition phase is characterized by strong to very strong and regular contractions, 8 to 10 cm dilation, and duration of 20 to 40 minutes.

Maternal nutritional status is an especially significant factor of the many that influence the outcome of pregnancy. Why is this the case? a. Maternal nutritional status is extremely difficult to adjust because of an individual's ingrained eating habits. b. Adequate nutrition is an important preventive measure for a variety of problems. c. Women love obsessing about their weight and diets. d. A woman's preconception weight becomes irrelevant.

ANS: B - Adequate nutrition is an important preventive measure for a variety of problems. Nutritional status draws so much attention not only for its effect on a healthy pregnancy and birth but also because significant changes are within relatively easy reach. Pregnancy is a time when many women are motivated to learn about adequate nutrition and make changes to their diet that will benefit their baby. Pregnancy is not the time to begin a weight loss diet. Clients and their caregivers should still be concerned with appropriate weight gain.

Ideally, when should prenatal care begin? a. Before the first missed menstrual period b. After the first missed menstrual period c. After the second missed menstrual period d. After the third missed menstrual period

ANS: B - After the first missed menstrual period Prenatal care should begin soon after the first missed menstrual period. This offers the greatest opportunities to ensure the health of the expectant mother and her infant. Prenatal care before missing the first menstrual period is too early. It is unlikely the woman is even aware of the pregnancy. Ideally, prenatal visits should begin soon after the first period is missed. Beginning prenatal care after the third missed menstrual period is too late. The woman will have completed the first trimester by that time.

Which sign of a potential complication is the most important for the nurse to share with the client? a. Constipation b. Alteration in the pattern of fetal movement c. Heart palpitations d. Edema in the ankles and feet at the end of the day

ANS: B - Alteration in the pattern of fetal movement An alteration in the pattern or amount of fetal movement may indicate fetal jeopardy. Constipation, heart palpitations, and ankle and foot edema are normal discomforts of pregnancy that occur in the second and third trimesters.

Which statement concerning the third stage of labor is correct? a. The placenta eventually detaches itself from a flaccid uterus. b. An expectant or active approach to managing this stage of labor reduces the risk of complications. c. It is important that the dark, roughened maternal surface of the placenta appears before the shiny fetal surface. d. The major risk for women during the third stage is a rapid heart rate.

ANS: B - An expectant or active approach to managing this stage of labor reduces the risk of complications. Active management facilitates placental separation and expulsion, reducing the risk of complications. The placenta cannot detach itself from a flaccid (relaxed) uterus. Which surface of the placenta comes out first is not clinically important. The major risk for women during the third stage of labor is postpartum hemorrhaging.

A 31-year-old woman believes that she may be pregnant. She took an over-the-counter (OTC) pregnancy test 1 week ago after missing her period; the test was positive. During her assessment interview, the nurse inquires about the woman's last menstrual period and asks whether she is taking any medications. The client states that she takes medicine for epilepsy. She has been under considerable stress lately at work and has not been sleeping well. Her physical examination does not indicate that she is pregnant. She has an ultrasound scan, which confirms that she is not pregnant. What is the most likely cause of the false-positive pregnancy test result? a. The pregnancy test was taken too early. b. Anticonvulsant medications may cause the false-positive test result. c. The woman has a fibroid tumor. d. She has been under considerable stress and has a hormone imbalance.

ANS: B - Anticonvulsant medications may cause the false-positive test result. Anticonvulsants may cause false-positive pregnancy test results. OTC pregnancy tests use enzyme-linked immunosorbent assay (ELISA) technology, which can yield positive results as soon as 4 days after implantation. Implantation occurs 6 to 10 days after conception. If the woman were pregnant, then she would be into her third week at this point (having missed her period 1 week ago). Fibroid tumors do not produce hormones and have no bearing on human chorionic gonadotropin (hCG) pregnancy tests. Although stress may interrupt normal hormone cycles (menstrual cycles), it does not affect hCG levels or produce positive pregnancy test results.

The maternity nurse is cognizant of what important structure and function of the placenta? a. As the placenta widens, it gradually thins to allow easier passage of air and nutrients. b. As one of its early functions, the placenta acts as an endocrine gland. c. The placenta is able to keep out most potentially toxic substances, such as cigarette smoke, to which the mother is exposed. d. Optimal blood circulation is achieved through the placenta when the woman is lying on her back or standing.

ANS: B - As one of its early functions, the placenta acts as an endocrine gland. The placenta produces four hormones necessary to maintain the pregnancy: hCG, hPL, estrogen, and progesterone. The placenta widens until 20 weeks of gestation and continues to grow thicker. Toxic substances such as nicotine and carbon monoxide readily cross the placenta into the fetus. Optimal circulation occurs when the woman is lying on her side.

To prevent gastrointestinal (GI) upset, when should a pregnant client be instructed to take the recommended iron supplements? a. On a full stomach b. At bedtime c. After eating a meal d. With milk

ANS: B - At bedtime Iron supplements taken at bedtime may reduce GI upset and should be taken at bedtime if abdominal discomfort occurs when iron supplements are taken between meals. Iron supplements are best absorbed if they are taken when the stomach is empty. Bran, tea, coffee, milk, and eggs may reduce absorption.

Which statement related to fetal positioning during labor is correct and important for the nurse to understand? a. Position is a measure of the degree of descent of the presenting part of the fetus through the birth canal. b. Birth is imminent when the presenting part is at +4 to +5 cm below the spine. c. The largest transverse diameter of the presenting part is the suboccipitobregmatic diameter. d. Engagement is the term used to describe the beginning of labor.

ANS: B - Birth is imminent when the presenting part is at +4 to +5 cm below the spine. The station of the presenting part should be noted at the beginning of labor to determine the rate of descent. Position is the relationship of the presenting part of the fetus to the four quadrants of the mother's pelvis; station is the measure of degree of descent. The largest diameter is usually the biparietal diameter. The suboccipitobregmatic diameter is the smallest, although one of the most critical. Engagement often occurs in the weeks just before labor in nulliparous women and before or during labor in multiparous women.

A client states that she plans to breastfeed her newborn infant. What guidance would be useful for this new mother? a. The mother's intake of vitamin C, zinc, and protein can now be lower than during pregnancy. b. Caffeine consumed by the mother accumulates in the infant, who may be unusually active and wakeful. c. Critical iron and folic acid levels must be maintained. d. Lactating women can go back to their prepregnant caloric intake.

ANS: B - Caffeine consumed by the mother accumulates in the infant, who may be unusually active and wakeful. A lactating woman needs to avoid consuming too much caffeine. Vitamin C, zinc, and protein levels need to be moderately higher during lactation than during pregnancy. The recommendations for iron and folic acid are lower during lactation. Lactating women should consume approximately 500 kcal more than their prepregnancy intake, at least 1800 kcal daily overall

The nurse has received a report regarding a client in labor. The woman's last vaginal examination was recorded as 3 cm, 30%, and -2. What is the nurse's interpretation of this assessment? a. Cervix is effaced 3 cm and dilated 30%; the presenting part is 2 cm above the ischial spines. b. Cervix is dilated 3 cm and effaced 30%; the presenting part is 2 cm above the ischial spines. c. Cervix is effaced 3 cm and dilated 30%; the presenting part is 2 cm below the ischial spines. d. Cervix is dilated 3 cm and effaced 30%; the presenting part is 2 cm below the ischial spines.

ANS: B - Cervix is dilated 3 cm and effaced 30%; the presenting part is 2 cm above the ischial spines. The sterile vaginal examination is recorded as centimeters of cervical dilation, percentage of cervical dilation, and the relationship of the presenting part to the ischial spines (either above or below). For this woman, the cervix is dilated 3 cm and effaced 30%, and the presenting part is 2 cm above the ischial spines. The first interpretation of this vaginal examination is incorrect; the cervix is dilated 3 cm and is 30% effaced. However, the presenting part is correct at 2 cm above the ischial spines. The remaining two interpretations of this vaginal examination are incorrect. Although the dilation and effacement are correct at 3 cm and 30%, the presenting part is actually 2 cm above the ischial spines.

The nurse is providing health education to a pregnant client regarding the cardiovascular system. Which information is correct and important to share? a. A pregnant woman experiencing disturbed cardiac rhythm, such as sinus arrhythmia, requires close medical and obstetric observation no matter how healthy she may appear otherwise. b. Changes in heart size and position and increases in blood volume create auditory changes from 20 weeks of gestation to term. c. Palpitations are twice as likely to occur in twin gestations. d. All of the above changes will likely occur.

ANS: B - Changes in heart size and position and increases in blood volume create auditory changes from 20 weeks of gestation to term. These auscultatory changes should be discernible after 20 weeks of gestation. A healthy woman with no underlying heart disease does not need any therapy. The maternal heart rate increases in the third trimester, but palpitations may not necessarily occur, let alone double. Auditory changes are discernible at 20 weeks of gestation.

During the first trimester, which of the following changes regarding her sexual drive should a client be taught to expect? a. Increased sexual drive, because of enlarging breasts b. Decreased sexual drive, because of nausea and fatigue c. No change in her sexual drive d. Increased sexual drive, because of increased levels of female hormones

ANS: B - Decreased sexual drive, because of nausea and fatigue A pregnant woman usually experiences a decrease, not an increase, in libido during the first trimester. Maternal physiologic changes, such as breast enlargement, nausea, fatigue, abdominal changes, perineal enlargement, leukorrhea, pelvic vasocongestion, and orgasmic responses, may affect sexuality and sexual expression. Libido may be depressed in the first trimester but often increases during the second and third trimesters. During pregnancy, the breasts may become enlarged and tender, which tends to interfere with coitus, thereby decreasing the desire to engage in sexual activity.

Which action is the highest priority for the nurse when educating a pregnant adolescent? a. Emphasize the need to eliminate common teenage snack foods because they are high in fat and sodium. b. Determine the weight gain needed to meet adolescent growth, and add 35 pounds. c. Suggest that she not eat at fast-food restaurants to avoid foods of poor nutritional value. d. Realize that most adolescents are unwilling to make dietary changes during pregnancy.

ANS: B - Determine the weight gain needed to meet adolescent growth, and add 35 pounds. Adolescents should gain in the upper range of the recommended weight gain. They also need to gain weight that would be expected for their own normal growth. Changes in the diet should be kept at a minimum. Snack foods can be included in moderation, and other foods can be added to make up for lost nutrients. Eliminating fast foods would make the adolescent appear different to her peers. The client should be taught to choose foods that add needed nutrients. Adolescents are willing to make changes; however, they still have the need to be similar to their peers.

Which statement best describes the rationale for the physiologic anemia that occurs during pregnancy? a. Physiologic anemia involves an inadequate intake of iron. b. Dilution of hemoglobin concentration occurs in pregnancy with physiologic anemia. c. Fetus establishes the iron stores. d. Decreased production of erythrocytes occur.

ANS: B - Dilution of hemoglobin concentration occurs in pregnancy with physiologic anemia. When blood volume expansion is more pronounced and occurs earlier than the increase in red blood cells, the woman has physiologic anemia, which is the result of the dilution of hemoglobin concentration rather than inadequate hemoglobin. An inadequate intake of iron may lead to true anemia. The production of erythrocytes increases during pregnancy.

The nurse should be aware of what important information regarding systemic analgesics administered during labor? a. Systemic analgesics cross the maternal blood-brain barrier as easily as they do the fetal blood-brain barrier. b. Effects on the fetus and newborn can include decreased alertness and delayed sucking. c. Intramuscular (IM) administration is preferred over IV administration. d. IV patient-controlled analgesia (PCA) results in increased use of an analgesic.

ANS: B - Effects on the fetus and newborn can include decreased alertness and delayed sucking. The effects of analgesics depend on the specific drug administered, the dosage, and the timing. Systemic analgesics cross the fetal blood-brain barrier more readily than the maternal blood-brain barrier. IV administration is preferred over IM administration because the drug acts faster and more predictably. PCA results in a decrease in the use of an analgesic.

A woman who is gravida 3 para 2 arrives on the intrapartum unit. What is the most important nursing assessment at this time? a. Contraction pattern, amount of discomfort, and pregnancy history b. FHR, maternal vital signs, and the woman's nearness to birth c. Identification of ruptured membranes, woman's gravida and para, and her support person d. Last food intake, when labor began, and cultural practices the couple desires

ANS: B - FHR, maternal vital signs, and the woman's nearness to birth All options describe relevant intrapartum nursing assessments; however, this focused assessment has a priority. If the maternal and fetal conditions are normal and birth is not imminent, then other assessments can be performed in an unhurried manner; these include: gravida, para, support person, pregnancy history, pain assessment, last food intake, and cultural practices.

Which statement concerning neurologic and sensory development in the fetus is correct? a. Brain waves have been recorded on an electroencephalogram as early as the end of the first trimester (12 weeks of gestation). b. Fetuses respond to sound by 24 weeks of gestation and can be soothed by the sound of the mother's voice. c. Eyes are first receptive to light at 34 to 36 weeks of gestation. d. At term, the fetal brain is at least one third the size of an adult brain.

ANS: B - Fetuses respond to sound by 24 weeks of gestation and can be soothed by the sound of the mother's voice. Hearing develops early and is fully developed at birth. Brain waves have been recorded at week 8. Eyes are receptive to light at 28 weeks of gestation. The fetal brain is approximately one fourth the size of an adult brain

Through a vaginal examination, the nurse determines that a woman is 4 cm dilated. The external fetal monitor shows uterine contractions every to 4 minutes. The nurse reports this as what stage of labor? a. First stage, latent phase b. First stage, active phase c. First stage, transition phase d. Second stage, latent phase

ANS: B - First stage, active phase This maternal progress indicates that the woman is in the active phase of the first stage of labor. During the latent phase of the first stage of labor, the expected maternal progress is 0 to 3 cm dilation with contractions every 5 to 30 minutes. During the transition phase of the first stage of labor, the expected maternal progress is 8 to 10 cm dilation with contractions every 2 to 3 minutes. During the latent phase of the second stage of labor, the woman is completely dilated and experiences a restful period of "laboring down."

Which finding in the urinalysis of a pregnant woman is considered a variation of normal? a. Proteinuria b. Glycosuria c. Bacteria in the urine d. Ketones in the urine

ANS: B - Glycosuria Small amounts of glucose may indicate physiologic spilling. The presence of protein could indicate kidney disease or preeclampsia. Urinary tract infections are associated with bacteria in the urine. An increase in ketones indicates that the patient is exercising too strenuously or has an inadequate fluid and food intake

What is the rationale for the use of a blood patch after spinal anesthesia? a. Hypotension b. Headache c. Neonatal respiratory depression d. Loss of movement

ANS: B - Headache The subarachnoid block may cause a postspinal headache resulting from the loss of cerebrospinal fluid from the puncture in the dura. When blood is injected into the epidural space in the area of the dural puncture, it forms a seal over the hole to stop the leaking of cerebrospinal fluid. Hypotension is prevented by increasing fluid volume before the procedure. Neonatal respiratory depression is not an expected outcome with spinal anesthesia. Loss of movement is an expected outcome of spinal anesthesia

What should the laboring client who receives an opioid antagonist be told to expect? a. Her pain will decrease. b. Her pain will return. c. She will feel less anxious. d. She will no longer feel the urge to push.

ANS: B - Her pain will return Opioid antagonists such as naloxone (Narcan) promptly reverse the CNS-depressant effects of opioids. In addition, the antagonist counters the effect of the stress-induced levels of endorphins. An opioid antagonist is especially valuable if the labor is more rapid than expected and the birth is anticipated when the opioid is at its peak effect. The woman should be told that the pain that was relieved by the opioid analgesic will return with the administration of the opioid antagonist. Her pain level will increase rather than decrease. Opioid antagonists have no effect on anxiety levels. They are primarily administered to reverse the excessive CNS depression in the mother, newborn, or both. An opioid antagonist (e.g., naloxone) has no effect on the mother's urge or ability to push. The practice of giving lower doses of IV opioids has reduced the incidence and severity of opioid-induced CNS depression; therefore, opioid antagonists are used less frequently

A 3-year-old girl's mother is 6 months pregnant. What concern is this child most likely to verbalize? a. How the baby will get out? b. How will the baby eat? c. Will you die having the baby? d. What color eyes will the baby have?

ANS: B - How will the baby eat? By age 3 or 4 years, children like to be told the story of their own beginning and accept it being compared with the present pregnancy. They like to listen to the fetal heartbeat and feel the baby move. Sometimes they worry about how the baby is being fed and what it will wear. School-age children take a more clinical interest in their mother's pregnancy and may want to know "How did the baby get in there?" and "How will it get out?" Whether the child's mother will die does not tend to be the focus of her questions about the impending birth of a sibling. The baby's eye color does not tend to be the focus of children's questions about the impending birth of a sibling.

Assessment of a woman's nutritional status includes a diet history, medication regimen, physical examination, and relevant laboratory tests. Which finding might require consultation to a higher level of care? a. Oral contraceptive use may interfere with the absorption of iron. b. Illnesses that have created nutritional deficits, such as PKU, may require nutritional care before conception. c. The woman's socioeconomic status and educational level are not relevant to her examination; they are the province of the social worker. d. Testing for diabetes is the only nutrition-related laboratory test most pregnant women need.

ANS: B - Illnesses that have created nutritional deficits, such as PKU, may require nutritional care before conception. A registered dietitian can help with therapeutic diets. Oral contraceptive use may interfere with the absorption of folic acid. Iron deficiency can appear if placement of an intrauterine device (IUD) results in blood loss. A woman's finances can affect her access to good nutrition; her education (or lack thereof) can influence the nurse's teaching decisions. The nutrition-related laboratory test that pregnant women usually need is a screen for anemia

Which renal system adaptation is an anticipated anatomic change of pregnancy? a. Increased urinary output makes pregnant women less susceptible to urinary infections. b. Increased bladder sensitivity and then compression of the bladder by the enlarging uterus result in the urge to urinate even when the bladder is almost empty. c. Renal (kidney) function is more efficient when the woman assumes a supine position. d. Using diuretic agents during pregnancy can help keep kidney function regular.

ANS: B - Increased bladder sensitivity and then compression of the bladder by the enlarging uterus result in the urge to urinate even when the bladder is almost empty. Bladder sensitivity and then compression of the bladder by the uterus result in the urge to urinate more often, even when the bladder is almost empty. A number of anatomic changes in pregnancy make a woman more susceptible to urinary tract infections. Renal function is more efficient when the woman lies in the lateral recumbent position and is less efficient when she is supine. Diuretic use during pregnancy can overstress the system and cause problems.

According to professional standards (the Association of Women's Health, Obstetric and Neonatal Nurses [AWHONN], 2007), which action cannot be performed by the nonanesthetist registered nurse who is caring for a woman with epidural anesthesia? a. Monitoring the status of the woman and fetus b. Initiating epidural anesthesia c. Replacing empty infusion bags with the same medication and concentrate d. Stopping the infusion, and initiating emergency measures

ANS: B - Initiating epidural anesthesia Only qualified, licensed anesthesia care providers are permitted to insert a catheter, initiate epidural anesthesia, verify catheter placement, inject medication through the catheter, or alter the medication or medications including type, amount, or rate of infusion. The nonanesthetist nurse is permitted to monitor the status of the woman, the fetus, and the progress of labor. Replacement of the empty infusion bags or syringes with the same medication and concentration is permitted. If the need arises, the nurse may stop the infusion, initiate emergency measures, and remove the catheter if properly educated to do so. Complications can require immediate interventions. Nurses must be prepared to provide safe and effective care during an emergency situation

Which characteristic correctly matches the type of deceleration with its likely cause? a. Early deceleration—umbilical cord compression b. Late deceleration—uteroplacental insufficiency c. Variable deceleration—head compression d. Prolonged deceleration—unknown cause

ANS: B - Late deceleration—uteroplacental insufficiency Late deceleration is caused by uteroplacental insufficiency. Early deceleration is caused by head compression. Variable deceleration is caused by umbilical cord compression. Prolonged deceleration has a variety of either benign or critical causes.

A woman who has a history of sexual abuse may have a number of traumatic memories triggered during labor. She may fight the labor process and react with pain or anger. The nurse can implement a number of care measures to help her client view the childbirth experience in a positive manner. Which intervention is key for the nurse to use while providing care? a. Tell the client to relax and that it won't hurt much. b. Limit the number of procedures that invade her body. c. Reassure the client that, as the nurse, you know what is best. d. Allow unlimited care providers to be with the client.

ANS: B - Limit the number of procedures that invade her body. The number of invasive procedures such as vaginal examinations, internal monitoring, and IV therapy should be limited as much as possible. The nurse should always avoid words and phrases that may result in the client's recalling the phrases of her abuser (i.e., "Relax, this won't hurt" or "Just open your legs"). The woman's sense of control should be maintained at all times. The nurse should explain procedures at the client's pace and wait for permission to proceed. Protecting the client's environment by providing privacy and limiting the number of staff who observe the client will help to make her feel safe

Which statement is the best rationale for assessing the maternal vital signs between uterine contractions? a. During a contraction, assessing the fetal heart rate is the priority. b. Maternal circulating blood volume temporarily increases during contractions. c. Maternal blood flow to the heart is reduced during contractions. d. Vital signs taken during contractions are not accurate.

ANS: B - Maternal circulating blood volume temporarily increases during contractions. During uterine contractions, blood flow to the placenta temporarily stops, causing a relative increase in the mother's blood volume, which, in turn, temporarily increases blood pressure and slows the pulse. Monitoring fetal responses to the contractions is important; however, this question concerns the maternal vital signs. Maternal blood flow is increased during a contraction. Vital signs are altered by contractions but are considered accurate for that period

What type of cultural concern is the most likely deterrent to many women seeking prenatal care? a. Religion b. Modesty c. Ignorance d. Belief that physicians are evil

ANS: B - Modesty A concern for modesty is a deterrent to many women seeking prenatal care. For some women, exposing body parts, especially to a man, is considered a major violation of their modesty. Many cultural variations are found in prenatal care. Even if the prenatal care described is familiar to a woman, some practices may conflict with the beliefs and practices of a subculture group to which she belongs.

What is the correct term describing the slight overlapping of cranial bones or shaping of the fetal head during labor? a. Lightening b. Molding c. Ferguson reflex d. Valsalva maneuver

ANS: B - Molding Molding also permits adaptation to various diameters of the maternal pelvis. Lightening is the mother's sensation of decreased abdominal distention, which usually occurs the week before labor. The Ferguson reflex is the contraction urge of the uterus after the stimulation of the cervix. The Valsalva maneuver describes conscious pushing during the second stage of labor.

The nurse working with pregnant clients must seek to gain understanding of the process whereby women accept their pregnancy. Which statement regarding this process is most accurate? a. Nonacceptance of the pregnancy very often equates to a rejection of the child. b. Mood swings are most likely the result of worries about finances and a changed lifestyle, as well as profound hormonal changes. c. Ambivalent feelings during pregnancy are usually only expressed in emotionally immature or very young mothers. d. Conflicts such as not wanting to be pregnant or childrearing and career-related decisions need not be addressed during pregnancy because they will naturally resolve themselves after birth.

ANS: B - Mood swings are most likely the result of worries about finances and a changed lifestyle, as well as profound hormonal changes. Mood swings are natural and are likely to affect every woman to some degree. A woman may dislike being pregnant, refuse to accept it, and still love and accept the child. Ambivalent feelings about pregnancy are normal for the mature or immature woman and for the younger or older woman. Conflicts such as not wanting to be pregnant or childrearing and career-related decisions need to be resolved. The baby ends the pregnancy but not all the issues.

Developing a realistic birth plan with the pregnant woman regarding her care is important for the nurse. How would the nurse explain the major advantage of nonpharmacologic pain management? a. Greater and more complete pain relief is possible. b. No side effects or risks to the fetus are involved. c. The woman will remain fully alert at all times. d. Labor will likely be more rapid.

ANS: B - No side effects or risks to the fetus are involved. Because nonpharmacologic pain management does not include analgesics, adjunct drugs, or anesthesia, it is harmless to the mother and the fetus. However, pain relief is lessened with nonpharmacologic pain management during childbirth. Although the woman's alertness is not altered by medication, the increase in pain may decrease alertness. Pain management may or may not alter the length of labor. At times when pain is decreased, the mother relaxes and labor progresses at a quicker pace.

During labor a fetus displays an average FHR of 135 beats per minute over a 10-minute period. Which statement best describes the status of this fetus? a. Bradycardia b. Normal baseline heart rate c. Tachycardia d. Hypoxia

ANS: B - Normal baseline heart rate The baseline FHR is measured over 10 minutes; a normal range is 110 to 160 beats per minute. Bradycardia is a FHR less than 110 beats per minute for 10 minutes or longer. Tachycardia is a FHR higher than 160 beats per minutes for 10 minutes or longer. Hypoxia is an inadequate supply of oxygen; no indication of hypoxia exists with a baseline FHR in the normal range

Which statement regarding the probable signs of pregnancy is most accurate? a. Determined by ultrasound b. Observed by the health care provider c. Reported by the client d. Confirmed by diagnostic tests

ANS: B - Observed by the health care provider Probable signs are those detected through trained examination. Fetal visualization is a positive sign of pregnancy. Presumptive signs are those reported by the client. The term diagnostic tests is open for interpretation. To actually diagnose pregnancy, one would have to see positive signs of pregnancy.

A pregnant woman at 18 weeks of gestation calls the clinic to report that she has been experiencing occasional backaches of mild-to-moderate intensity. Which intervention should the nurse recommend? a. Kegel exercises b. Pelvic rock exercises c. Softer mattress d. Bed rest for 24 hours

ANS: B - Pelvic rock exercises Pelvic rock exercises may help stretch and strengthen the abdominal and lower back muscles and relieve low back pain. Stretching and other exercises to relieve back pain should be performed several times a day. Kegel exercises increase the tone of the pelvic area, not the back. A softer mattress may not provide the support needed to maintain proper alignment of the spine and may contribute to back pain.

What is the correct terminology for the nerve block that provides anesthesia to the lower vagina and perineum? a. Epidural b. Pudendal c. Local d. Spinal block

ANS: B - Pudendal A pudendal block anesthetizes the lower vagina and perineum to provide anesthesia for an episiotomy and the use of low forceps, if needed. An epidural provides anesthesia for the uterus, perineum, and legs. A local provides anesthesia for the perineum at the site of the episiotomy. A spinal block provides anesthesia for the uterus, perineum, and down the legs.

Which statement regarding the structure and function of the placenta is correct? a. Produces nutrients for fetal nutrition b. Secretes both estrogen and progesterone c. Forms a protective, impenetrable barrier to microorganisms such as bacteria and viruses d. Excretes prolactin and insulin

ANS: B - Secretes both estrogen and progesterone As one of its early functions, the placenta acts as an endocrine gland, producing four hormones necessary to maintain the pregnancy and to support the embryo or fetus: human chorionic gonadotropin (hCG), human placental lactogen (hPL), estrogen, and progesterone. The placenta does not produce nutrients. It functions as a means of metabolic exchange between the maternal and fetal blood supplies. Many bacteria and viruses can cross the placental membrane.

Anxiety is commonly associated with pain during labor. Which statement regarding anxiety is correct? a. Even mild anxiety must be treated. b. Severe anxiety increases tension, increases pain, and then, in turn, increases fear and anxiety, and so on. c. Anxiety may increase the perception of pain, but it does not affect the mechanism of labor. d. Women who have had a painful labor will have learned from the experience and have less anxiety the second time because of increased familiarity.

ANS: B - Severe anxiety increases tension, increases pain, and then, in turn, increases fear and anxiety, and so on. Anxiety and pain reinforce each other in a negative cycle that will slow the progress of labor. Mild anxiety is normal for a woman in labor and likely needs no special treatment other than the standard reassurances. Anxiety increases muscle tension and ultimately can sufficiently build to slow the progress of labor. Unfortunately, an anxious, painful first labor is likely to carry over, through expectations and memories, into an anxious and painful experience in the second pregnancy.

Many clients are concerned about the increased levels of mercury in fish and may be reluctant to include this source of nutrients in their diet. What is the best advice for the nurse to provide? a. Canned white tuna is a preferred choice. b. Shark, swordfish, and mackerel should be avoided. c. Fish caught in local waterways is the safest. d. Salmon and shrimp contain high levels of mercury.

ANS: B - Shark, swordfish, and mackerel should be avoided. As a precaution, the pregnant client should avoid eating shark, swordfish, and mackerel, as well as the less common tilefish. High levels of mercury can harm the developing nervous system of the fetus. Assisting the client in understanding the differences between numerous sources of mercury is essential for the nurse. A pregnant client may eat as much as 12 ounces a week of canned light tuna; however, canned white, albacore, or tuna steaks contain higher levels of mercury and should be limited to no more than 6 ounces per week. Pregnant women and mothers of young children should check with local advisories about the safety of fish caught by families and friends in nearby bodies of water. If no information is available, then these fish sources should be avoided, limited to less than 6 ounces per week, or the only fish consumed that week. Commercially caught fish that is low in mercury includes salmon, shrimp, pollock, or catfish. The pregnant client may eat up to 12 ounces of commercially caught fish per week. Additional information on levels of mercury in commercially caught fish is available at www.cfsan.fda.gov.

The musculoskeletal system adapts to the changes that occur throughout the pregnancy. Which musculoskeletal alteration should the client expect? a. Her center of gravity will shift backward. b. She will have increased lordosis. c. She will have increased abdominal muscle tone. d. She will notice decreased mobility of her pelvic joints.

ANS: B - She will have increased lordosis. An increase in the normal lumbosacral curve (lordosis) develops, and a compensatory curvature in the cervicodorsal region develops to help her maintain balance. The center of gravity shifts forward. She will have decreased abdominal muscle tone and will notice increased mobility of her pelvic joints.

A client is seen at the clinic at 14 weeks of gestation for a follow-up appointment. At which level does the nurse expect to palpate the fundus? a. Nonpalpable above the symphysis at 14 weeks of gestation b. Slightly above the symphysis pubis c. At the level of the umbilicus d. Slightly above the umbilicus

ANS: B - Slightly above the symphysis pubis In normal pregnancies, the uterus grows at a predictable rate. It may be palpated above the symphysis pubis sometime between the 12th and 14th weeks of pregnancy. As the uterus grows, it may be palpated above the symphysis pubis sometime between the 12th and 14th weeks of pregnancy. At 14 weeks, the uterus is not yet at the level of the umbilicus. The fundus is not palpable above the umbilicus until 22 to 24 weeks of gestation.

Which time-based description of a stage of development in pregnancy is correct? a. Viability—22 to 37 weeks of gestation since the last menstrual period (assuming a fetal weight greater than 500 g) b. Term—pregnancy from the beginning of 38 weeks of gestation to the end of 42 weeks of gestation c. Preterm—pregnancy from 20 to 28 weeks of gestation d. Postdate—pregnancy that extends beyond 38 weeks of gestation

ANS: B - Term—pregnancy from the beginning of 38 weeks Term is 38 to 42 weeks of gestation. Viability is the ability of the fetus to live outside the uterus before coming to term, or 22 to 24 weeks since the last menstrual period. Preterm is 20 to 37 weeks of gestation. Postdate or postterm is a pregnancy that extends beyond 42 weeks of gestation or what is considered the limit of full term

During the physical examination of a client beginning prenatal care, which initial action is most important for the nurse to perform? a. Only women who show physical signs or meet the sociologic profile should be assessed for physical abuse. b. The client should empty her bladder before the pelvic examination. c. The distribution, amount, and quality of body hair are of no particular importance. d. The size of the uterus is discounted in the initial examination because it will be increasing in size during the second trimester.

ANS: B - The client should empty her bladder before the pelvic examination. The nurse should instruct the client to empty her bladder. An empty bladder facilitates the examination and also provides an opportunity to obtain a urine sample for a number of tests. All women should be assessed for a history of physical abuse, particularly because the likelihood of abuse increases during pregnancy. Noting body hair is important because body hair reflects nutritional status, endocrine function, and hygiene. Particular attention is paid to the size of the uterus because it is an indication of the duration of gestation.

The client is instructed to place her thumb and forefinger on the areola and gently press inward. What is the purpose of this exercise? a. To check the sensitivity of the nipples b. To determine whether the nipple is everted or inverted c. To calculate the adipose buildup in the abdomen d. To see whether the fetus has become inactive

ANS: B - To determine whether the nipple is everted or inverted Sometimes known as the pinch test, this exercise is used to determine whether the nipple is everted or inverted. Nipples must be everted to allow breastfeeding. The pinch does not determine the level of sensitivity of the nipples, nor is it not used to determine the level of adipose tissue in the abdomen. Fetal activity is not determined by using the pinch test.

Cardiac output increases from 30% to 50% by the 32nd week of pregnancy. What is the rationale for this change? a. To compensate for the decreased renal plasma flow b. To provide adequate perfusion of the placenta c. To eliminate metabolic wastes of the mother d. To prevent maternal and fetal dehydration

ANS: B - To provide adequate perfusion of the placenta The primary function of increased vascular volume is to transport oxygen and nutrients to the fetus via the placenta. Renal plasma flow increases during pregnancy. Assisting with pulling metabolic wastes from the fetus for maternal excretion is one purpose of the increased vascular volume.

After an emergency birth, the nurse encourages the woman to breastfeed her newborn. What is the primary purpose of this activity? a. To facilitate maternal-newborn interaction b. To stimulate the uterus to contract c. To prevent neonatal hypoglycemia d. To initiate the lactation cycle

ANS: B - To stimulate the uterus to contract Stimulation of the nipples through breastfeeding or manual stimulation causes the release of oxytocin and prevents maternal hemorrhage. Breastfeeding facilitates maternal-newborn interaction, but it is not the primary reason a woman is encouraged to breastfeed after an emergency birth. The primary intervention for preventing neonatal hypoglycemia is thermoregulation. Cold stress can result in hypoglycemia. The woman is encouraged to breastfeed after an emergency birth to stimulate the release of oxytocin, which prevents hemorrhaging. Breastfeeding is encouraged to initiate the lactation cycle, but it is not the primary reason for this activity after an emergency birth.

What is the rationale for the administration of an oxytocic (e.g., Pitocin, Methergine) after expulsion of the placenta? a. To relieve pain b. To stimulate uterine contraction c. To prevent infection d. To facilitate rest and relaxation

ANS: B - To stimulate uterine contraction Oxytocics stimulate uterine contractions, which reduce blood loss after the third stage of labor. Oxytocics are not used to treat pain, do not prevent infection, and do not facilitate rest and relaxation.

What is the primary difference between the labor of a nullipara and that of a multipara? a. Amount of cervical dilation b. Total duration of labor c. Level of pain experienced d. Sequence of labor mechanisms

ANS: B - Total duration of labor In a first-time pregnancy, the descent is usually slow but steady; in subsequent pregnancies, the descent is more rapid, resulting in a shorter duration of labor. Cervical dilation is the same for all labors. The level of pain is individual to the woman, not to the number of labors she has experienced. The sequence of labor mechanisms is the same with all labors.

What three measures should the nurse implement to provide intrauterine resuscitation? a. Call the provider, reposition the mother, and perform a vaginal examination. b. Turn the client onto her side, provide oxygen (O2) via face mask, and increase intravenous (IV) fluids. c. Administer O2 to the mother, increase IV fluids, and notify the health care provider. d. Perform a vaginal examination, reposition the mother, and provide O2 via face mask.

ANS: B - Turn the client onto her side, provide oxygen (O2) via face mask, and increase intravenous (IV) fluids. Basic interventions for the management of any abnormal FHR pattern include administering O2 via a nonrebreather face mask at a rate of 8 to 10 L/min, assisting the woman onto a side-lying (lateral) position, and increasing blood volume by increasing the rate of the primary IV infusion. The purpose of these interventions is to improve uterine blood flow and intervillous space blood flow and to increase maternal oxygenation and cardiac output. The term intrauterine resuscitation is sometimes used to refer to these interventions. If these interventions do not quickly resolve the abnormal FHR issue, then the primary provider should be immediately notified

Which statement regarding multifetal pregnancy is incorrect? a. The expectant mother often develops anemia because the fetuses have a greater demand for iron. b. Twin pregnancies come to term with the same frequency as single pregnancies. c. The mother should be counseled to increase her nutritional intake and gain more weight. d. Backache and varicose veins often are more pronounced with a multifetal pregnancy

ANS: B - Twin pregnancies come to term with the same frequency as single pregnancies. Twin pregnancies often end in prematurity. Serious efforts should be made to bring the pregnancy to term. A woman with a multifetal pregnancy often develops anemia, suffers more or worse backache, and needs to gain more weight. Counseling is needed to help her adjust to these conditions.

What is the most likely cause for variable FHR decelerations? a. Altered fetal cerebral blood flow b. Umbilical cord compression c. Uteroplacental insufficiency d. Fetal hypoxemia

ANS: B - Umbilical cord compression Variable FHR decelerations can occur at any time during the uterine contracting phase and are caused by compression of the umbilical cord. Altered fetal cerebral blood flow results in early decelerations in the FHR. Uteroplacental insufficiency results in late decelerations in the FHR. Fetal hypoxemia initially results in tachycardia and then bradycardia if hypoxia continues

Under which circumstance would it be unnecessary for the nurse to perform a vaginal examination? a. Admission to the hospital at the start of labor b. When accelerations of the FHR are noted c. On maternal perception of perineal pressure or the urge to bear down d. When membranes rupture

ANS: B - When accelerations of the FHR are noted An accelerated FHR is a positive sign; therefore, a vaginal examination would not be necessary. A vaginal examination should be performed when the woman is admitted to the hospital, when she perceives perineal pressure or the urge to bear down, when her membranes rupture, when a significant change in her uterine activity has occurred, or when variable decelerations of the FHR are noted.

Of which physiologic alteration of the uterus during pregnancy is it important for the nurse to alert the patient? a. Lightening occurs near the end of the second trimester as the uterus rises into a different position. b. Woman's increased urinary frequency in the first trimester is the result of exaggerated uterine antireflexion caused by softening. c. Braxton Hicks contractions become more painful in the third trimester, particularly if the woman tries to exercise. d. Uterine souffle is the movement of the fetus.

ANS: B - Woman's increased urinary frequency in the first trimester is the result of exaggerated uterine antireflexion caused by softening. The softening of the lower uterine segment is called the Hegar sign. In this position, the uterine fundus presses on the bladder, causing urinary frequency that is a normal change of pregnancy. Lightening occurs in the last 2 weeks of pregnancy, when the fetus descends. Braxton Hicks contractions become more defined in the final trimester but are not painful. Walking or exercise usually causes them to stop. The uterine souffle is the sound made by blood in the uterine arteries; it can be heard with a fetal stethoscope.

Which client might be well advised to continue condom use during intercourse throughout her pregnancy? a. Unmarried pregnant women b. Women at risk for acquiring or transmitting STIs c. All pregnant women d. Women at risk for candidiasis

ANS: B - Women at risk for acquiring or transmitting STIs The objective of safer sex is to provide prophylaxis against the acquisition and transmission of STIs. Because these diseases may be transmitted to the woman and then to her fetus, condom use is recommended throughout the pregnancy if the woman is at risk for acquiring an STI. Pregnant women are encouraged to practice safer sex behaviors. An unmarried pregnant woman may be in a monogamous relationship and not require the use of a condom. The client should be educated as to what may place both herself and her fetus at risk. Any pregnant woman can develop candidiasis, which is an infection not related to condom use.

The pancreas forms in the foregut during the 5th to 8th week of gestation. A client with poorly controlled gestational diabetes asks the nurse what the effects of her condition will be on the fetus. What is the best response by the nurse? Poorly controlled maternal gestational diabetes will: a. produce fetal hypoglycemia. b. result in a macrocosmic fetus. c. result in a microcosmic fetus. d. enhance lung maturation.

ANS: B - result in a macrocosmic fetus. Insulin is produced by week 20 of gestation. In the fetus of a mother with uncontrolled diabetes, maternal hypoglycemia produces fetal hypoglycemia and macrocosmia results. Hyperinsulinemia blocks lung maturation, placing the neonate at risk for respiratory distress.

A pregnant woman at 25 weeks of gestation tells the nurse that she dropped a pan last week and her baby jumped at the noise. Which response by the nurse is most accurate? a. "That must have been a coincidence; babies can't respond like that." b. "The fetus is demonstrating the aural reflex." c. "Babies respond to sound starting at approximately 24 weeks of gestation." d. "Let me know if it happens again; we need to report that to your midwife."

ANS: C - "Babies respond to sound starting at approximately 24 weeks of gestation." Babies respond to external sound starting at approximately 24 weeks of gestation. Acoustic stimulations can evoke a fetal heart rate response. There is no such thing as an aural reflex. The last statement is inappropriate and may cause undue psychologic alarm to the client.

A woman's cousin gave birth to an infant with a congenital heart anomaly. The woman asks the nurse when such anomalies occur during development. Which response by the nurse is most accurate? a. "We don't really know when such defects occur." b. "It depends on what caused the defect." c. "Defects occur between the third and fifth weeks of development." d. "They usually occur in the first 2 weeks of development."

ANS: C - "Defects occur between the third and fifth weeks of development." The cardiovascular system is the first organ system to function in the developing human. Blood vessel and blood formation begins in the third week, and the heart is developmentally complete in the fifth week. "We don't really know when such defects occur" is an inaccurate statement. Regardless of the cause, the heart is vulnerable during its period of development—in the third to fifth weeks; therefore, the statement, "They usually occur in the first 2 weeks of development" is inaccurate

Which statement by the client would lead the nurse to believe that labor has been established? a. "I passed some thick, pink mucus when I urinated this morning." b. "My bag of waters just broke." c. "The contractions in my uterus are getting stronger and closer together." d. "My baby dropped, and I have to urinate more frequently now."

ANS: C - "The contractions in my uterus are getting stronger and closer together." Regular, strong contractions with the presence of cervical change indicate that the woman is experiencing true labor. Although the loss of the mucous plug (operculum) often occurs during the first stage of labor or before the onset of labor, it is not the indicator of true labor. Spontaneous rupture of membranes often occurs during the first stage of labor; however, it is not an indicator of true labor. The presenting part of the fetus typically becomes engaged in the pelvis at the onset of labor but is not the indicator of true labor

Which statement by the client will assist the nurse in determining whether she is in true labor as opposed to false labor? a. "I passed some thick, pink mucus when I urinated this morning." b. "My bag of waters just broke." c. "The contractions in my uterus are getting stronger and closer together." d. "My baby dropped, and I have to urinate more frequently now."

ANS: C - "The contractions in my uterus are getting stronger and closer together." Regular, strong contractions with the presence of cervical change indicate that the woman is experiencing true labor. The loss of the mucous plug (operculum) often occurs during the first stage of labor or before the onset of labor, but it is not the indicator of true labor. Spontaneous rupture of membranes (ROM) often occurs during the first stage of labor, but it is not the indicator of true labor. The presenting part of the fetus typically becomes engaged in the pelvis at the onset of labor, but this is not the indicator of true labor

A client arrives for her initial prenatal examination. This is her first child. She asks the nurse, "How does my baby get air inside my uterus?" What is the correct response by the nurse? a. "The baby's lungs work in utero to exchange oxygen and carbon dioxide." b. "The baby absorbs oxygen from your blood system." c. "The placenta provides oxygen to the baby and excretes carbon dioxide into your bloodstream." d. "The placenta delivers oxygen-rich blood through the umbilical artery to the baby's abdomen."

ANS: C - "The placenta provides oxygen to the baby and excretes carbon dioxide into your bloodstream." The placenta delivers oxygen-rich blood through the umbilical vein, not the artery, to the fetus and excretes carbon dioxide into the maternal bloodstream. The fetal lungs do not function as respiratory gas exchange in utero. The baby does not simply absorb oxygen from a woman's blood system; rather, blood and gas transport occur through the placenta.

A pregnant woman at 10 weeks of gestation jogs three or four times per week. She is concerned about the effect of the exercise on the fetus. Which guidance should the nurse provide? a. "You don't need to modify your exercising any time during your pregnancy." b. "Stop exercising because it will harm the fetus." c. "You may find that you need to modify your exercise to walking later in your pregnancy, around the seventh month." d. "Jogging is too hard on your joints; switch to walking now."

ANS: C - "You may find that you need to modify your exercise to walking later in your pregnancy, around the seventh month." Typically, running should be replaced with walking around the seventh month of pregnancy. The nurse should inform the woman that she may need to reduce her exercise level as the pregnancy progresses. Physical activity promotes a feeling of well-being in pregnant women. It improves circulation, promotes relaxation and rest, and counteracts boredom. Simple measures should be initiated to prevent injuries, such as warm-up and stretching exercises to prepare the joints for more strenuous exercise.

A 27-year-old pregnant woman had a preconceptual body mass index (BMI) of 19. What is this client's total recommended weight gain during pregnancy? a. 20 kg (44 lb) b. 16 kg (35 lb) c. 12.5 kg (27.5 lb) d. 10 kg (22 lb)

ANS: C - 12.5 kg (27.5 lb) This woman has a normal BMI and should gain 11.5 to 16 kg during her pregnancy. A weight gain of 20 kg (44 lb) is unhealthy for most women; a weight gain of 16 kg (35 lb) is at the high end of the range of weight this woman should gain in her pregnancy; and a weight gain of 10 kg (22 lb) is appropriate for an obese woman. This woman has a normal BMI, which indicates that her weight is average

If a client's normal prepregnancy diet contains 45 g of protein daily, how many more grams of protein should she consume per day during pregnancy? a. 5 b. 10 c. 25 d. 30

ANS: C - 25 The recommended intake of protein for the pregnant woman is 70 g. Therefore, additional protein intakes of 5, 10, or 15 g would be inadequate to meet protein needs during pregnancy. A protein intake of 30 g is more than would be necessary and would add extra calories.

At a routine prenatal visit, the nurse explains the development of the fetus to her client. At approximately ____ weeks of gestation, lecithin is forming on the alveolar surfaces, the eyelids open, and the fetus measures approximately 27 cm crown to rump and weighs approximately 1110 g. The client is how many weeks of gestation at today's visit? a. 20 b. 24 c. 28 d. 30

ANS: C - 28 These milestones in human development occur at 28 weeks of gestation. These milestones have not occurred by 20 or 24 weeks of gestation but have been reached before 30 weeks of gestation.

The measurement of lecithin in relation to sphingomyelin (lecithin/sphingomyelin [L/S] ratio) is used to determine fetal lung maturity. Which ratio reflects fetal maturity of the lungs? a. 1.4:1 b. 1.8:1 c. 2:1 d. 1:1

ANS: C - 2:1 The L/S ratio indicates a 2:1 ratio of lecithin to sphingomyelin, which is an indicator of fetal lung maturity and occurs at approximately the middle of the third trimester. L/S ratios of 1.4:1, 1.8:1, and 1:1 each indicate immaturity of the fetal lungs

Which information related to a prolonged deceleration is important for the labor nurse to understand? a. Prolonged decelerations present a continuing pattern of benign decelerations that do not require intervention. b. Prolonged decelerations constitute a baseline change when they last longer than 5 minutes. c. A disruption to the fetal oxygen supply causes prolonged decelerations. d. Prolonged decelerations require the customary fetal monitoring by the nurse.

ANS: C - A disruption to the fetal oxygen supply causes prolonged decelerations. Prolonged decelerations are caused by a disruption in the fetal oxygen supply. They usually begin as a reflex response to hypoxia. If the disruption continues, then the fetal cardiac tissue, itself, will become hypoxic, resulting in direct myocardial depression of the FHR. Prolonged decelerations can be caused by prolonged cord compression, uteroplacental insufficiency, or perhaps sustained head compression. Prolonged decelerations lasting longer than 10 minutes are considered a baseline change that may require intervention. A prolonged deceleration is a visually apparent decrease (may be either gradual or abrupt) in the FHR of at least 15 beats per minute below the baseline and lasting longer than 2 minutes but shorter than 10 minutes. Nurses should immediately notify the physician or nurse-midwife and initiate appropriate treatment of abnormal patterns when they see prolonged decelerations.

A nulliparous woman has just begun the latent phase of the second stage of her labor. The nurse should anticipate which behavior? a. A nulliparous woman will experience a strong urge to bear down. b. Perineal bulging will show. c. A nulliparous woman will remain quiet with her eyes closed between contractions. d. The amount of bright red bloody show will increase.

ANS: C - A nulliparous woman will remain quiet with her eyes closed between contractions. The woman is able to relax and close her eyes between contractions as the fetus passively descends. The woman may be very quiet during this phase. During the latent phase of the second stage of labor, the urge to bear down is often absent or only slight during the acme of the contractions. Perineal bulging occurs during the transition phase of the second stage of labor, not at the beginning of the second stage. An increase in bright red bloody show occurs during the descent phase of the second stage of labor

A woman in labor is breathing into a mouthpiece just before the start of her regular contractions. As she inhales, a valve opens and gas is released. She continues to inhale the gas slowly and deeply until the contraction starts to subside. When the inhalation stops, the valve closes. Which statement regarding this procedure is correct? a. The application of nitrous oxide gas is not often used anymore. b. An inhalation of gas is likely to be used in the second stage of labor, not during the first stage. c. An application of nitrous oxide gas is administered for pain relief. d. The application of gas is a prelude to a cesarean birth.

ANS: C - An application of nitrous oxide gas is administered for pain relief. A mixture of nitrous oxide with oxygen in a low concentration can be used in combination with other nonpharmacologic and pharmacologic measures for pain relief. This procedure is still commonly used in Canada and in the United Kingdom. Nitrous oxide inhaled in a low concentration will reduce but not eliminate pain during the first and second stages of labor. Nitrous oxide inhalation is not generally used before a caesarean birth. Nitrous oxide does not appear to depress uterine contractions or cause adverse reactions in the newborn.

Which statement accurately describes the centering model of care? a. Group sessions begin with the first prenatal visit. b. Blood pressure (BP), weight, and urine dipsticks are obtained by the nurse at each visit. c. Approximately 8 to 12 women are placed in each gestational-age cohort group. d. Outcomes are similar to traditional prenatal care.

ANS: C - Approximately 8 to 12 women are placed in each gestational-age cohort group. Gestational-age cohorts comprise the groups, with approximately 8 to 12 women in each group. The groups remain intact throughout the pregnancy. Individual follow-up visits are scheduled as needed. Group sessions begin at 12 to 16 weeks of gestation and end with an early postpartum visit. Before the group sessions, the client has an individual assessment, physical examination, and history. At the beginning of each group meeting, clients measure their own BP, weight, and urine dips and enter these findings in their record. Fetal heart rate assessment and fundal height are obtained by the nurse. Results evaluating this approach have been very promising. In a recent study of adolescent clients, the number of LBW infants decreased and breastfeeding rates increased.

The nurse is performing an initial assessment of a client in labor. What is the appropriate terminology for the relationship of the fetal body parts to one another? a. Lie b. Presentation c. Attitude d. Position

ANS: C - Attitude Attitude is the relationship of the fetal body parts to one another. Lie is the relationship of the long axis (spine) of the fetus to the long axis (spine) of the mother. Presentation refers to the part of the fetus that enters the pelvic inlet first and leads through the birth canal during labor at term. Position is the relationship of the presenting part of the fetus to the four quadrants of the mother's pelvis.

Which statement is not an expected outcome for the client who attends a reputable childbirth preparation program? a. Childbirth preparation programs increase the woman's sense of control. b. Childbirth preparation programs prepare a support person to help during labor. c. Childbirth preparation programs guarantee a pain-free childbirth. d. Childbirth preparation programs teach distraction techniques.

ANS: C - Childbirth preparation programs guarantee a pain-free childbirth. All methods try to increase a woman's sense of control, prepare a support person, and train the woman in physical conditioning, which includes breathing techniques. These programs cannot, and reputable ones do not, promise a pain-free childbirth. Increasing a woman's sense of control is the goal of all childbirth preparation methods. Preparing a support person to help in labor is a vitally important component of any childbirth education program. The coach may learn how to touch a woman's body to detect tense and contracted muscles. The woman then learns how to relax in response to the gentle stroking by the coach. Distraction techniques are a form of care that are effective to some degree in relieving labor pain and are taught in many childbirth programs. These distractions include imagery, feedback relaxation, and attention-focusing behaviors

Which description of the phases of the second stage of labor is most accurate? a. Latent phase: Feeling sleepy; fetal station 2+ to 4+; duration of 30 to 45 minutes b. Active phase: Overwhelmingly strong contractions; Ferguson reflux activated; duration of 5 to 15 minutes c. Descent phase: Significant increase in contractions; Ferguson reflux activated; average duration varies d. Transitional phase: Woman "laboring down"; fetal station 0; duration of 15 minutes

ANS: C - Descent phase: Significant increase in contractions; Ferguson reflux activated; average duration varies The descent phase begins with a significant increase in contractions; the Ferguson reflex is activated, and the duration varies, depending on a number of factors. The latent phase is the lull or "laboring down" period at the beginning of the second stage and lasts 10 to 30 minutes on average. The second stage of labor has no active phase. The transition phase is the final phase in the second stage of labor; contractions are strong and painful.

The uterine contractions of a woman early in the active phase of labor are assessed by an internal uterine pressure catheter (IUPC). The uterine contractions occur every 3 to 4 minutes and last an average of 55 to 60 seconds. They are becoming more regular and are moderate to strong. Based on this information, what would a prudent nurse do next? a. Immediately notify the woman's primary health care provider. b. Prepare to administer an oxytocic to stimulate uterine activity. c. Document the findings because they reflect the expected contraction pattern for the active phase of labor. d. Prepare the woman for the onset of the second stage of labor.

ANS: C - Document the findings because they reflect the expected contraction pattern for the active phase of labor. The nurse is responsible for monitoring the uterine contractions to ascertain whether they are powerful and frequent enough to accomplish the work of expelling the fetus and the placenta. In addition, the nurse documents these findings in the client's medical record. This labor pattern indicates that the client is in the active phase of the first stage of labor. Nothing indicates a need to notify the primary health care provider at this time. Oxytocin augmentation is not needed for this labor pattern; this contraction pattern indicates that the woman is in active labor. Her contractions will eventually become stronger, last longer, and come closer together during the transition phase of the first stage of labor. The transition phase precedes the second stage of labor, or delivery of the fetus.

Which consideration is essential for the nurse to understand regarding follow-up prenatal care visits? a. The interview portions become more intensive as the visits become more frequent over the course of the pregnancy b. Monthly visits are scheduled for the first trimester, every 2 weeks for the second trimester, and weekly for the third trimester. c. During the abdominal examination, the nurse should be alert for supine hypotension. d. For pregnant women, a systolic BP of 130 mm Hg and a diastolic BP of 80 mm Hg is sufficient to be considered hypertensive.

ANS: C - During the abdominal examination, the nurse should be alert for supine hypotension. The woman lies on her back during the abdominal examination, possibly compressing the vena cava and aorta, which can cause a decrease in BP and a feeling of faintness. The interview portion of the follow-up examinations is less extensive than in the initial prenatal visits, during which so much new information must be gathered. Monthly visits are routinely scheduled for the first and second trimesters; visits increase to every 2 weeks at week 28 and to once a week at week 36. For pregnant women, hypertension is defined as a systolic BP of 140 mm Hg or higher and a diastolic BP of 90 mm Hg or higher.

Nurses should be cognizant of what regarding the mechanism of labor? a. Seven critical movements must progress in a more or less orderly sequence. b. Asynclitism is sometimes achieved by means of the Leopold's maneuver. c. Effects of the forces determining descent are modified by the shape of the woman's pelvis and the size of the fetal head. d. At birth, the baby is said to achieve "restitution"; that is, a return to the C-shape of the womb.

ANS: C - Effects of the forces determining descent are modified by the shape of the woman's pelvis and the size of the fetal head. The size of the maternal pelvis and the ability of the fetal head to mold also affect the process. The seven identifiable movements of the mechanism of labor simultaneously occur in combinations, not in precise sequences. Asynclitism is the deflection of the baby's head; the Leopold's maneuver is a means of judging descent by palpating the mother's abdomen. Restitution is the rotation of the baby's head after the infant is born.

Which action is correct when palpation is used to assess the characteristics and pattern of uterine contractions? a. Placing the hand on the abdomen below the umbilicus and palpating uterine tone with the fingertips b. Determining the frequency by timing from the end of one contraction to the end of the next contraction c. Evaluating the intensity by pressing the fingertips into the uterine fundus d. Assessing uterine contractions every 30 minutes throughout the first stage of labor

ANS: C - Evaluating the intensity by pressing the fingertips into the uterine fundus The nurse or primary health care provider may assess uterine activity by palpating the fundal section of the uterus using the fingertips. Many women may experience labor pain in the lower segment of the uterus, which may be unrelated to the firmness of the contraction detectable in the uterine fundus. The frequency of uterine contractions is determined by palpating from the beginning of one contraction to the beginning of the next contraction. Assessment of uterine activity is performed in intervals based on the stage of labor. As labor progresses, this assessment is performed more frequently

The first 1 to 2 hours after birth is sometimes referred to as what? a. Bonding period b. Third stage of labor c. Fourth stage of labor d. Early postpartum period

ANS: C - Fourth stage of labor The first 2 hours of the birth are a critical time for the mother and her baby and is often called the fourth stage of labor. Maternal organs undergo their initial readjustment to a nonpregnant state. The third stage of labor lasts from the birth of the baby to the expulsion of the placenta. Bonding will occur over a much longer period, although it may be initiated during the fourth stage of labor.

A woman's position is an important component of the labor progress. Which guidance is important for the nurse to provide to the laboring client? a. The supine position, which is commonly used in the United States, increases blood flow. b. The laboring client positioned on her hands and knees ("all fours" position) is hard on the woman's back. c. Frequent changes in position help relieve fatigue and increase the comfort of the laboring client. d. In a sitting or squatting position, abdominal muscles of the laboring client will have to work harder.

ANS: C - Frequent changes in position help relieve fatigue and increase the comfort of the laboring client. Frequent position changes relieve fatigue, increase comfort, and improve circulation. Blood flow can be compromised in the supine position; any upright position benefits cardiac output. The "all fours" position is used to relieve backache in certain situations. In a sitting or squatting position, the abdominal muscles work in greater harmony with uterine contractions

Where is the point of maximal intensity (PMI) of the FHR located? a. Usually directly over the fetal abdomen b. In a vertex position, heard above the mother's umbilicus c. Heard lower and closer to the midline of the mother's abdomen as the fetus descends and internally rotates d. In a breech position, heard below the mother's umbilicus

ANS: C - Heard lower and closer to the midline of the mother's abdomen as the fetus descends and internally rotates Nurses should be prepared for the shift. The PMI of the FHR is usually directly over the fetal back. In a vertex position, the PMI of the FHR is heard below the mother's umbilicus. In a breech position, it is heard above the mother's umbilicus.

Nurses with an understanding of cultural differences regarding likely reactions to pain may be better able to help their clients. Which clients may initially appear very stoic but then become quite vocal as labor progresses until late in labor, when they become more vocal and request pain relief? a. Chinese b. Arab or Middle Eastern c. Hispanic d. African-American

ANS: C - Hispanic Hispanic women may be stoic early in labor but more vocal and ready for medications later. Chinese women may not show reactions to pain. Medical interventions must be offered more than once. Arab or Middle Eastern women may be vocal in response to labor pain from the start; they may prefer pain medications. African-American women may openly express pain; the use of medications for pain is more likely to vary with the individual

Dental care during pregnancy is an important component of good prenatal care. Which instruction regarding dental health should the nurse provide? a. Regular brushing and flossing may not be necessary during early pregnancy because it may stimulate the woman who is already nauseated to vomit. A cleaning is all that is necessary. b. Dental surgery, in particular, is contraindicated during pregnancy and should be delayed until after delivery. c. If dental treatment is necessary, then the woman will be most comfortable with it in the second trimester. d. If a woman has dental anxiety, then dental care may interfere with the expectant mother's need to practice conscious relaxation and to prepare for labor.

ANS: C - If dental treatment is necessary, then the woman will be most comfortable with it in the second trimester. The second trimester is the best time for dental treatment because the woman will be able to sit most comfortably in the dental chair. Dental care, such as brushing with a fluoride toothpaste, is especially important during pregnancy. Periodontal disease has been linked to both preterm labor and low-birth-weight (LBW) infants. Emergency dental surgery is permissible; however, the mother must clearly understand the risks and benefits. Conscious relaxation is useful and may even help the woman get through any dental appointments, but it is not a reason to avoid them.

Which minerals and vitamins are usually recommended as a supplement in a pregnant client's diet? a. Fat-soluble vitamins A and D b. Water-soluble vitamins C and B6 c. Iron and folate d. Calcium and zinc

ANS: C - Iron and folate Iron should generally be supplemented, and folic acid supplements are often needed because folate is so important in pregnancy. Fat-soluble vitamins should be supplemented as a medical prescription, as vitamin D might be for lactose-intolerant women. Water-soluble vitamin C is sometimes naturally consumed in excess; vitamin B6 is prescribed only if the woman has a very poor diet; and zinc is sometimes supplemented. Most women get enough calcium.

Which adaptation of the maternal-fetal exchange of oxygen occurs in response to uterine contraction? a. The maternal-fetal exchange of oxygen and waste products continues except when placental functions are reduced. b. This maternal-fetal exchange increases as the blood pressure decreases. c. It diminishes as the spiral arteries are compressed. d. This exchange of oxygen and waste products is not significantly affected by contractions.

ANS: C - It diminishes as the spiral arteries are compressed. Uterine contractions during labor tend to decrease circulation through the spiral electrodes and subsequent perfusion through the intervillous space. The maternal blood supply to the placenta gradually stops with contractions. The exchange of oxygen and waste products decreases. The exchange of oxygen and waste products is affected by contractions.

The nurse is providing education to a client regarding the normal changes of the breasts during pregnancy. Which statement regarding these changes is correct? a. The visibility of blood vessels that form an intertwining blue network indicates full function of the Montgomery tubercles and possibly an infection of the tubercles. b. The mammary glands do not develop until 2 weeks before labor. c. Lactation is inhibited until the estrogen level declines after birth. d. Colostrum is the yellowish oily substance used to lubricate the nipples for breastfeeding.

ANS: C - Lactation is inhibited until the estrogen level declines after birth. Lactation is inhibited until after birth. The visible blue network of blood vessels is a normal outgrowth of a richer blood supply. The mammary glands are functionally complete by midpregnancy. Colostrum is a creamy white-to-yellow premilk fluid that can be expressed from the nipples before birth.

Pregnant adolescents are at greater risk for decreased BMI and "fad" dieting with which condition? a. Obesity b. Gestational diabetes c. Low-birth-weight babies d. High-birth-weight babies

ANS: C - Low-birth-weight babies Adolescents tend to have lower BMIs. In addition, the fetus and the still-growing mother appear to compete for nutrients. These factors, along with inadequate weight gain, lend themselves to a higher incidence of low-birth-weight babies. Obesity is associated with a higher-than-normal BMI. Unless the teenager has type 1 diabetes, an adolescent with a low BMI is less likely to develop gestational diabetes. High-birth-weight or large-for-gestational age (LGA) babies are most often associated with gestational diabetes.

Which statement regarding the development of the respiratory system is a high priority for the nurse to understand? a. The respiratory system does not begin developing until after the embryonic stage. b. The infant's lungs are considered mature when the L/S ratio is 1:1, at approximately 32 weeks of gestation. c. Maternal hypertension can reduce maternal-placental blood flow, accelerating lung maturity. d. Fetal respiratory movements are not visible on ultrasound scans until at least 16 weeks of gestation.

ANS: C - Maternal hypertension can reduce maternal-placental blood flow, accelerating lung maturity. A reduction in placental blood flow stresses the fetus, increases blood levels of corticosteroids, and thus accelerates lung maturity. The development of the respiratory system begins during the embryonic phase and continues into childhood. The infant's lungs are considered mature when the L/S ratio is 2:1, at approximately 35 weeks of gestation. Lung movements have been visualized on ultrasound scans at 11 weeks of gestation.

The nurse observes a sudden increase in variability on the ERM tracing. Which class of medications may cause this finding? a. Narcotics b. Barbiturates c. Methamphetamines d. Tranquilizers

ANS: C - Methamphetamines Narcotics, barbiturates, and tranquilizers may be causes of decreased variability; whereas methamphetamines may cause increased variability.

A woman in labor has just received an epidural block. What is the most important nursing intervention at this time? a. Limit parenteral fluids. b. Monitor the fetus for possible tachycardia. c. Monitor the maternal blood pressure for possible hypotension. d. Monitor the maternal pulse for possible bradycardia.

ANS: C - Monitor the maternal blood pressure for possible hypotension. The most important nursing intervention for a woman who has received an epidural block is for the nurse to monitor the maternal blood pressure frequently for signs of hypotension. IV fluids are increased for a woman receiving an epidural to prevent hypotension. The nurse also observes for signs of fetal bradycardia and monitors for signs of maternal tachycardia, secondary to hypotension

A laboring woman has received meperidine (Demerol) intravenously (IV), 90 minutes before giving birth. Which medication should be available to reduce the postnatal effects of meperidine on the neonate? a. Fentanyl (Sublimaze) b. Promethazine (Phenergan) c. Naloxone (Narcan) d. Nalbuphine (Nubain)

ANS: C - Naloxone (Narcan) An opioid antagonist can be given to the newborn as one part of the treatment for neonatal narcosis, which is a state of central nervous system (CNS) depression in the newborn produced by an opioid. Opioid antagonists, such as naloxone (Narcan), can promptly reverse the CNS depressant effects, especially respiratory depression. Fentanyl (Sublimaze), promethazine (Phenergan), and nalbuphine (Nubain) do not act as opioid antagonists to reduce the postnatal effects of meperidine on the neonate.

A labor and delivery nurse should be cognizant of which information regarding how the fetus moves through the birth canal? a. Fetal attitude describes the angle at which the fetus exits the uterus. b. Of the two primary fetal lies, the horizontal lie is that in which the long axis of the fetus is parallel to the long axis of the mother. c. Normal attitude of the fetus is called general flexion. d. Transverse lie is preferred for vaginal birth.

ANS: C - Normal attitude of the fetus is called general flexion. The normal attitude of the fetus is called general flexion. The fetal attitude is the relationship of the fetal body parts to each one another. The horizontal lie is perpendicular to the mother; in the longitudinal (or vertical) lie, the long axes of the fetus and the mother are parallel. Vaginal birth cannot occur if the fetus stays in a transverse lie.

A woman arrives at the clinic for a pregnancy test. Her last menstrual period (LMP) was February 14, 2015. What is the client's expected date of birth (EDB)? a. September 17, 2015 b. November 7, 2015 c. November 21, 2015 d. December 17, 2015

ANS: C - November 21, 2015 Using the Nägele's rule, the EDB is calculated by subtracting 3 months from the month of the LMP and adding 7 days + 1 year to the day of the LMP. Therefore, with an LMP of February 14, 2015, her due date is November 21, 2015. September 17, 2015, is too short a period to complete a normal pregnancy. Using the Nägele's rule, an EDB of November 7, 2015, is 2 weeks early. December 17, 2015, is almost a month past the correct EDB.

Which clinical finding in a primiparous client at 32 weeks of gestation might be an indication of anemia? a. Ptyalism b. Pyrosis c. Pica d. Decreased peristalsis

ANS: C - Pica Pica (a desire to eat nonfood substances) is an indication of iron deficiency and should be evaluated. Cravings include ice, clay, and laundry starch. Ptyalism (excessive salivation), pyrosis (heartburn), and decreased peristalsis are normal findings.

While obtaining a diet history, the nurse might be told that the expectant mother has cravings for ice chips, cornstarch, and baking soda. Which nutritional problem does this behavior indicate? a. Preeclampsia b. Pyrosis c. Pica d. Purging

ANS: C - Pica The consumption of foods low in nutritional value or of nonfood substances (e.g., dirt, laundry starch) is called pica. Preeclampsia is a vasospastic disease process encountered after 20 weeks of gestation. Characteristics of preeclampsia include increasing hypertension, proteinuria, and hemoconcentration. Pyrosis is a burning sensation in the epigastric region, otherwise known as heartburn. Purging refers to self-induced vomiting after consuming large quantities of food

In which clinical situation would the nurse most likely anticipate a fetal bradycardia? a. Intraamniotic infection b. Fetal anemia c. Prolonged umbilical cord compression d. Tocolytic treatment using terbutaline

ANS: C - Prolonged umbilical cord compression Fetal bradycardia can be considered a later sign of fetal hypoxia and is known to occur before fetal death. Bradycardia can result from placental transfer of drugs, prolonged compression of the umbilical cord, maternal hypothermia, and maternal hypotension. Intraamniotic infection, fetal anemia, and tocolytic treatment using terbutaline would most likely result in fetal tachycardia.

When assessing the fetus using Leopold's maneuvers, the nurse feels a round, firm, and movable fetal part in the fundal portion of the uterus and a long, smooth surface in the mother's right side close to midline. What is the position of the fetus? a. ROA b. LSP c. RSA d. LOA

ANS: C - RSA Fetal position is denoted with a three-letter abbreviation. The first letter indicates the presenting part in either the right or the left side of the maternal pelvis. The second letter indicates the anatomic presenting part of the fetus. The third letter stands for the location of the presenting part in relationship to the anterior, posterior, or transverse portion of the maternal pelvis. Palpation of a round, firm fetal part in the fundal portion of the uterus would be the fetal head, indicating that the fetus is in a breech position with the sacrum as the presenting part in the maternal pelvis. Palpation of the fetal spine along the mother's right side denotes the location of the presenting part in the mother's pelvis. The ability to palpate the fetal spine indicates that the fetus is anteriorly positioned in the maternal pelvis. This fetus is anteriorly positioned in the right side of the maternal pelvis with the sacrum as the presenting part. RSA is the correct three-letter abbreviation to indicate this fetal position. ROA denotes a fetus that is anteriorly positioned in the right side of the maternal pelvis with the occiput as the presenting part. LSP describes a fetus that is posteriorly positioned in the left side of the pelvis with the sacrum as the presenting part. A fetus that is LOA would be anteriorly positioned in the left side of the pelvis with the occiput as the presenting part.

What is the primary rationale for the thorough drying of the infant immediately after birth? a. Stimulates crying and lung expansion b. Removes maternal blood from the skin surface c. Reduces heat loss from evaporation d. Increases blood supply to the hands and feet

ANS: C - Reduces heat loss from evaporation Infants are wet with amniotic fluid and blood at birth, and this accelerates evaporative heat loss. The primary purpose of drying the infant is to prevent heat loss. Although rubbing the infant stimulates crying, it is not the main reason for drying the infant. This process does not remove all the maternal blood.

A woman has requested an epidural for her pain. She is 5 cm dilated and 100% effaced. The baby is in a vertex position and is engaged. The nurse increases the woman's IV fluid for a preprocedural bolus. The nurse reviews her laboratory values and notes that the woman's hemoglobin is 12 g/dl, hematocrit is 38%, platelets are 67,000, and white blood cells (WBCs) are 12,000/mm3 Which factor would contraindicate an epidural for this woman? a. She is too far dilated. b. She is anemic. c. She has thrombocytopenia. d. She is septic.

ANS: C - She has thrombocytopenia. The platelet count indicates a coagulopathy, specifically, thrombocytopenia (low platelets), which is a contraindication to epidural analgesia and anesthesia. Typically, epidural analgesia and anesthesia are used in the laboring woman when a regular labor pattern has been achieved, as evidenced by progressive cervical change. The laboratory values show that the woman's hemoglobin and hematocrit levels are in the normal range and show a slight increase in the WBC count that is not uncommon in laboring women.

The labor and delivery nurse is preparing a client who is severely obese (bariatric) for an elective cesarean birth. Which piece of specialized equipment will not likely be needed when providing care for this pregnant woman? a. Extra-long surgical instruments b. Wide surgical table c. Temporal thermometer d. Increased diameter blood pressure cuff

ANS: C - Temporal thermometer Obstetricians today are seeing an increasing number of morbidly obese pregnant women weighing 400, 500, and 600 pounds. To manage their conditions and to meet their logistical needs, a new medical subspecialty, bariatric obstetrics, has arisen. Extra-wide blood pressure cuffs, scales that can accommodate up to 880 pounds, and extra-wide surgical tables designed to hold the weight of these women are used. Special techniques for ultrasound examination and longer surgical instruments for cesarean birth are also required. A temporal thermometer can be used for a pregnant client of any size.

During the initial visit with a client who is beginning prenatal care, which action should be the highest priority for the nurse? a. The first interview is a relaxed, get-acquainted affair during which the nurse gathers some general impressions of his or her new client. b. If the nurse observed handicapping conditions, he or she should be sensitive and not inquire about them because the client will do that in her own time. c. The nurse should be alert to the appearance of potential parenting problems, such as depression or lack of family support. d. Because of legal complications, the nurse should not ask about illegal drug use; that is left to the physician.

ANS: C - The nurse should be alert to the appearance of potential parenting problems, such as depression or lack of family support. Besides these potential problems, the nurse needs to be alert to the woman's attitude toward keeping regular health care appointments. If the client lacks insurance, then the nurse may be able to direct her to resources that provide assistance for pregnant women (i.e., Women, Infants, and Children [WIC]; Medicaid). The initial interview needs to be planned, purposeful, and focused on specific content. A lot of ground must be covered. The nurse must be sensitive to special problems; he or she should inquire because discovering individual needs is important. A client with a chronic or handicapping condition might forget to mention it because she has adapted to it. Obtaining information on drug use is important and can be confidentially done. Actual testing for drug use requires the client's consent.

What is a distinct advantage of external EFM? a. The ultrasound transducer can accurately measure short-term variability and beat-to-beat changes in the FHR. b. The tocotransducer can measure and record the frequency, regularity, intensity, and approximate duration of uterine contractions. c. The tocotransducer is especially valuable for measuring uterine activity during the first stage of labor. d. Once correctly applied by the nurse, the transducer need not be repositioned even when the woman changes positions.

ANS: C - The tocotransducer is especially valuable for measuring uterine activity during the first stage of labor. The tocotransducer is valuable for measuring uterine activity during the first stage of labor and is especially true when the membranes are intact. Short-term variability and beat-to-beat changes cannot be measured with this technology. The tocotransducer cannot measure and record the intensity of uterine contractions. The transducer must be repositioned when the woman or the fetus changes position

What is the primary role of the nonpregnant partner during pregnancy? a. To provide financial support b. To protect the pregnant woman from "old wives' tales" c. To support and nurture the pregnant woman d. To make sure the pregnant woman keeps prenatal appointments

ANS: C - To support and nurture the pregnant woman The partner's primary role in pregnancy is to nurture the pregnant woman and respond to her feelings of vulnerability. Although financial support is important, it is not the partner's primary role in pregnancy. Protecting the pregnant woman from "old wives' tales" is not the partner's role. The woman's partner can encourage the client to keep all appointments; however, this is not the most important role during the pregnancy.

A 25-year-old gravida 3, para 2 client gave birth to a 9-pound, 7-ounce boy, 4 hours ago after augmentation of labor with oxytocin (Pitocin). She presses her call light, and asks for her nurse right away, stating "I'm bleeding a lot." What is the most likely cause of postpartum hemorrhaging in this client? a. Retained placental fragments b. Unrepaired vaginal lacerations c. Uterine atony d. Puerperal infection

ANS: C - Uterine atony This woman gave birth to a macrosomic infant after oxytocin augmentation. Combined with these risk factors, uterine atony is the most likely cause of bleeding 4 hours after delivery. Although retained placental fragments may cause postpartum hemorrhaging, it is typically detected within the first hour after delivery of the placenta and is not the most likely cause of the hemorrhaging in this woman. Although unrepaired vaginal lacerations may also cause bleeding, it typically occurs in the period immediately after birth. Puerperal infection can cause subinvolution and subsequent bleeding that is, however, typically detected 24 hours postpartum.

The nurse who provides care to clients in labor must have a thorough understanding of the physiologic processes of maternal hypotension. Which outcome might occur if the interventions for maternal hypotension are inadequate? a. Early FHR decelerations b. Fetal arrhythmias c. Uteroplacental insufficiency d. Spontaneous rupture of membranes

ANS: C - Uteroplacental insufficiency Low maternal blood pressure reduces placental blood flow during uterine contractions, resulting in fetal hypoxemia. Maternal hypotension does not result in early FHR decelerations nor is it associated with fetal arrhythmias. Spontaneous rupture of membranes is not a result of maternal hypotension.

The nurse providing care for a high-risk laboring woman is alert for late FHR decelerations. Which clinical finding might be the cause for these late decelerations? a. Altered cerebral blood flow b. Umbilical cord compression c. Uteroplacental insufficiency d. Meconium fluid

ANS: C - Uteroplacental insufficiency Uteroplacental insufficiency results in late FHR decelerations. Altered fetal cerebral blood flow results in early FHR decelerations. Umbilical cord compression results in variable FHR decelerations. Meconium-stained fluid may or may not produce changes in the FHR, depending on the gestational age of the fetus and whether other causative factors associated with fetal distress are present

A pregnant woman's diet consists almost entirely of whole grain breads and cereals, fruits, and vegetables. Which dietary requirement is the nurse most concerned about? a. Calcium b. Protein c. Vitamin B12 d. Folic acid

ANS: C - Vitamin B12 A pregnant woman's diet is consistent with that followed by a strict vegetarian (vegan). Vegans consume only plant products. Because vitamin B12 is found in foods of animal origin, this diet is deficient in vitamin B12. Depending on the woman's food choices, a pregnant woman's diet may be adequate in calcium. Protein needs can be sufficiently met by a vegetarian diet. The nurse should be more concerned with the woman's intake of vitamin B12 attributable to her dietary restrictions. Folic acid needs can be met by enriched bread products.

Some pregnant clients may complain of changes in their voice and impaired hearing. What should the nurse explain to the client concerning these findings? a. Voice changes are caused by decreased estrogen levels. b. Displacement of the diaphragm results in thoracic breathing. c. Voice changes and impaired hearing are due to the results of congestion and swelling of the upper respiratory tract. d. Increased blood volume causes changes in the voice.

ANS: C - Voice changes and impaired hearing are due to the results of congestion and swelling of the upper respiratory tract. Although the diaphragm is displaced and the volume of blood is increased, neither causes changes in the voice nor impairs hearing. The key is that estrogen levels increase, not decrease, which causes the upper respiratory tract to become more vascular, which produces swelling and congestion in the nose and ears and therefore voice changes and impaired hearing.

Which nursing assessment indicates that a woman who is in second-stage labor is almost ready to give birth? a. Fetal head is felt at 0 station during vaginal examination b. Bloody mucous discharge increases. c. Vulva bulges and encircles the fetal head. d. Membranes rupture during a contraction.

ANS: C - Vulva bulges and encircles the fetal head. A bulging vulva that encircles the fetal head describes crowning, which occurs shortly before birth. Birth of the head occurs when the station is +4. A 0 station indicates engagement. Bloody show occurs throughout the labor process and is not an indication of an imminent birth. ROM can occur at any time during the labor process and does not indicate an imminent birth.

Which nursing assessment indicates that a woman who is in second-stage labor is almost ready to give birth? a. Fetal head is felt at 0 station during the vaginal examination. b. Bloody mucous discharge increases. c. Vulva bulges and encircles the fetal head. d. Membranes rupture during a contraction.

ANS: C - Vulva bulges and encircles the fetal head. During the active pushing (descent) phase, the woman has strong urges to bear down as the presenting part of the fetus descends and presses on the stretch receptors of the pelvic floor. The vulva stretches and begins to bulge, encircling the fetal head. Birth of the head occurs when the station is +4. A 0 station indicates engagement. Bloody show occurs throughout the labor process and is not an indication of an imminent birth. Rupture of membranes can occur at any time during the labor process and does not indicate an imminent birth

Which clinical finding indicates that the client has reached the second stage of labor? a. Amniotic membranes rupture. b. Cervix cannot be felt during a vaginal examination. c. Woman experiences a strong urge to bear down. d. Presenting part of the fetus is below the ischial spines.

ANS: C - Woman experiences a strong urge to bear down. During the descent phase of the second stage of labor, the woman may experience an increase in the urge to bear down. The ROM has no significance in determining the stage of labor. The second stage of labor begins with full cervical dilation. Many women may have an urge to bear down when the presenting fetal part is below the level of the ischial spines. This urge can occur during the first stage of labor, as early as with 5 cm dilation.

What should the nurse be cognizant of concerning the client's reordering of personal relationships during pregnancy? a. Because of the special motherhood bond, a woman's relationship with her mother is even more important than with the father of the child. b. Nurses need not get involved in any sexual issues the couple has during pregnancy, particularly if they have trouble communicating them to each other. c. Women usually express two major relationship needs during pregnancy: feeling loved and valued and having the child accepted by the father. d. The woman's sexual desire is likely to be highest in the first trimester because of the excitement and because intercourse is physically easier.

ANS: C - Women usually express two major relationship needs during pregnancy: feeling loved and valued and having the child accepted by the father. Love and support help a woman feel better about her pregnancy. The most important person to the pregnant woman is usually the father of the child. Nurses can facilitate communication between partners about sexual matters if, as is common, they are nervous about expressing their worries and feelings to one another. The second trimester is the time when a woman's sense of well-being, along with certain physical changes, increases her desire for sex. Sexual desire is down in the first and third trimesters.

With regard to weight gain during pregnancy, the nurse should be aware of which important information? a. In pregnancy, the woman's height is not a factor in determining her target weight. b. Obese women may have their health concerns, but their risk of giving birth to a child with major congenital defects is the same as with women of normal weight. c. Women with inadequate weight gain have an increased risk of delivering a preterm infant with intrauterine growth restriction (IUGR). d. Greater than expected weight gain during pregnancy is almost always attributable to old-fashioned overeating.

ANS: C - Women with inadequate weight gain have an increased risk of delivering a preterm infant with intrauterine growth restriction (IUGR). IUGR is associated with women with inadequate weight gain. The primary factor in making a weight gain recommendation is the appropriateness of the prepregnancy weight for the woman's height. Obese women are twice as likely as women of normal weight to give birth to a child with major congenital defects. Overeating is only one of several likely causes.

A woman who is 14 weeks pregnant tells the nurse that she always had a glass of wine with dinner before she became pregnant. She has abstained during her first trimester and would like to know if it is safe for her to have a drink with dinner now. Which guidance should the nurse provide? a. "Since you're in your second trimester, there's no problem with having one drink with dinner." b. "One drink every night is too much. One drink three times a week should be fine." c. "Since you're in your second trimester, you can drink as much as you like." d. "Because no one knows how much or how little alcohol it takes to cause fetal problems, the best course is to abstain throughout your pregnancy."

ANS: D - "Because no one knows how much or how little alcohol it takes to cause fetal problems, the best course is to abstain throughout your pregnancy." The statement "Because no one knows how much or how little alcohol it takes to cause fetal problems, the best course is to abstain throughout your pregnancy" is accurate. A safe level of alcohol consumption during pregnancy has not yet been established. Although the consumption of occasional alcoholic beverages may not be harmful to the mother or her developing fetus, complete abstinence is strongly advised

A woman is 3 months pregnant. At her prenatal visit she tells the nurse that she does not know what is happening; one minute she is happy that she is pregnant and the next minute she cries for no reason. Which response by the nurse is most appropriate? a. "Don't worry about it; you'll feel better in a month or so." b. "Have you talked to your husband about how you feel?" c. "Perhaps you really don't want to be pregnant." d. "Hormone changes during pregnancy commonly result in mood swings."

ANS: D - "Hormone changes during pregnancy commonly result in mood swings." Explaining that hormone changes can result in mood swings is an accurate statement and the most appropriate response by the nurse. Telling the woman not to worry dismisses her concerns and is not the most appropriate response. Although the woman should be encouraged to share her feelings, asking if she has spoken to her husband about them is not the most appropriate response and does not provide her with a rationale for the psychosocial dynamics of her pregnancy. Suggesting that the woman does not want to be pregnant is completely inappropriate and deleterious to the psychologic well-being of the woman. Hormonal and metabolic adaptations often cause mood swings in pregnancy. The woman's responses are normal. She should be reassured about her feelings.

What represents a typical progression through the phases of a woman's establishing a relationship with the fetus? a. Accepts the fetus as distinct from herself—accepts the biologic fact of pregnancy—has feelings of caring and responsibility. b. Fantasizes about the child's gender and personality—views the child as part of herself—becomes introspective. c. Views the child as part of herself—has feelings of well-being—accepts the biologic fact of the pregnancy. d. "I am pregnant"—"I am going to have a baby"—"I am going to be a mother."

ANS: D - "I am pregnant"—"I am going to have a baby"—"I am going to be a mother." The woman first centers on herself as pregnant, then on the baby as an entity separate from herself, and then on her responsibilities as a mother. The expressions "I am pregnant," "I am going to have a baby," and "I am going to be a mother" sum up the progression through the three phases. In phase one, the woman views the child as part of herself and not as a separate being. This is only the first step of the progression through phases of attachment. Accepting the fetus as distinct from herself occurs during the second phase of emotional attachment. Fantasizing about the child's sex and personality based on fetal activity occurs during the third phase of attachment.

A woman has come to the clinic for preconception counseling because she wants to start trying to get pregnant. Which guidance should she expect to receive? a. "Discontinue all contraception now." b. "Lose weight so that you can gain more during pregnancy." c. "You may take any medications you have been regularly taking." d. "Make sure you include adequate folic acid in your diet."

ANS: D - "Make sure you include adequate folic acid in your diet." A healthy diet before conception is the best way to ensure that adequate nutrients are available for the developing fetus. A woman's folate or folic acid intake is of particular concern in the periconception period. Neural tube defects are more common in infants of women with a poor folic acid intake. Depending on the type of contraception that she has been using, discontinuing all contraception at this time may not be appropriate. Advising this client to lose weight now so that she can gain more during pregnancy is also not appropriate advice. Depending on the type of medications the woman is taking, continuing to take them regularly may not be appropriate

A pregnant couple has formulated a birth plan and is reviewing it with the nurse at an expectant parent's class. Which aspect of their birth plan should be considered potentially unrealistic and require further discussion with the nurse? a. "My husband and I have agreed that my sister will be my coach because he becomes anxious with regard to medical procedures and blood. He will be nearby and check on me every so often to make sure everything is okay." b. "We plan to use the techniques taught in the Lamaze classes to reduce the pain experienced during labor." c. "We want the labor and birth to take place in a birthing room. My husband will come in the minute the baby is born." d. "Regardless of the circumstances, we do not want the fetal monitor used during labor because it will interfere with movement and doing effleurage."

ANS: D - "Regardless of the circumstances, we do not want the fetal monitor used during labor because it will interfere with movement and doing effleurage." Because monitoring is essential to assess fetal well-being, fetal monitoring is not a factor that can be determined by the couple. The nurse should fully explain its importance. The option for intermittent electronic monitoring could be explored if this is a low-risk pregnancy and as long as labor is normally progressing. The birth plan is a tool with which parents can explore their childbirth options; however, the plan must be viewed as tentative. Having the woman's sister as her coach with her husband nearby is an acceptable request for a laboring woman. Using breathing techniques to alleviate pain is a realistic part of a birth plan. Not all fathers are able to be present during the birth; however, this couple has made a realistic plan that works for their specific situation

A client is in early labor, and her nurse is discussing the pain relief options she is considering. The client states that she wants an epidural "no matter what!" What is the nurse's best response? a. "I'll make sure you get your epidural." b. "You may only have an epidural if your physician allows it." c. "You may only have an epidural if you are going to deliver vaginally." d. "The type of analgesia or anesthesia used is determined, in part, by the stage of your labor and the method of birth."

ANS: D - "The type of analgesia or anesthesia used is determined, in part, by the stage of your labor and the method of birth." To avoid suppressing the progress of labor, pharmacologic measures for pain relief are generally not implemented until labor has advanced to the active phase of the first stage and the cervix is dilated approximately 4 to 5 cm. A plan of care is developed for each woman that addresses her particular clinical and nursing problems. The nurse collaborates with the primary health care provider and the laboring woman in selecting features of care relevant to the woman and her family. The decision whether to use an epidural to relieve labor pain is multifactorial. The nurse should not make a blanket statement guaranteeing the client one pharmacologic option over another until a complete history and physical examination has been obtained. A physician's order is required for pharmacologic options for pain management. However, expressing this requirement is not the nurse's best response. An epidural is an effective pharmacologic pain management option for many laboring women. It can also be used for anesthesia control if the woman undergoes an operative delivery

A woman who is 8 months pregnant asks the nurse, "Does my baby have any antibodies to fight infection?" What is the most appropriate response by the nurse? a. "Your baby has all the immunoglobulins necessary: immunoglobulin G (IgG), immunoglobulin M (IgM), and immunoglobulin A (IgA)." b. "Your baby won't receive any antibodies until he is born and you breastfeed him." c. "Your baby does not have any antibodies to fight infection." d. "Your baby has IgG and IgM." ANS: D - "Your baby has IgG and IgM." During the third trimester, IgG is the only immunoglobulin that crosses the placenta; it provides passive acquired immunity to specific bacterial toxins. However, the fetus produces IgM by the end of the first trimester. IgA immunoglobulins are not produced by the baby. Therefore, by the third trimester, the fetus has both IgG and IgM. Breastfeeding supplies the newborn infant with IgA.

ANS: D - "Your baby has IgG and IgM." During the third trimester, IgG is the only immunoglobulin that crosses the placenta; it provides passive acquired immunity to specific bacterial toxins. However, the fetus produces IgM by the end of the first trimester. IgA immunoglobulins are not produced by the baby. Therefore, by the third trimester, the fetus has both IgG and IgM. Breastfeeding supplies the newborn infant with IgA.

A new mother asks the nurse when the "soft spot" on her son's head will go away. What is the nurse's best response, based upon her understanding of when the anterior frontal closes? a. 2 months b. 8 months c. 12 months d. 18 months

ANS: D - 18 months The larger of the two fontanels, the anterior fontanel, closes by 18 months after birth. The posterior fontanel closes at 6 to 8 weeks. The remaining three options are too early for the anterior fontanel to close

Some of the embryo's intestines remain within the umbilical cord during the embryonic period. What is the rationale for this development of the gastrointestinal system? a. Umbilical cord is much larger at this time than it will be at the end of pregnancy. b. Intestines begin their development within the umbilical cord. c. Nutrient content of the blood is higher in this location. d. Abdomen is too small to contain all the organs while they are developing.

ANS: D - Abdomen is too small to contain all the organs while they are developing. The abdominal contents grow more rapidly than the abdominal cavity; therefore, part of their development takes place in the umbilical cord. By 10 weeks of gestation, the abdomen is large enough to contain them. Intestines begin their development within the umbilical cord but only because the liver and kidneys occupy most of the abdominal cavity. Blood supply is adequate in all areas.

Which definition of an acceleration in the fetal heart rate (FHR) is accurate? a. FHR accelerations are indications of fetal well-being when they are periodic. b. FHR accelerations are greater and longer in preterm gestations. c. FHR accelerations are usually observed with breech presentations when they are episodic. d. An acceleration in the FHR presents a visually apparent and abrupt peak.

ANS: D - An acceleration in the FHR presents a visually apparent and abrupt peak. Acceleration of the FHR is defined as a visually apparent abrupt (only to peak 30 seconds) increase in the FHR above the baseline rate. Periodic accelerations occur with uterine contractions and are usually observed with breech presentations. Episodic accelerations occur during fetal movement and are indications of fetal well-being. Preterm accelerations peak at 10 beats per minute above the baseline and last for at least 10 seconds.

Which characteristic of a uterine contraction is not routinely documented? a. Frequency: how often contractions occur b. Intensity: strength of the contraction at its peak c. Resting tone: tension in the uterine muscle d. Appearance: shape and height

ANS: D - Appearance: shape and height Uterine contractions are described in terms of frequency, intensity, duration, and resting tone. Appearance is not routinely charted.

When a nulliparous woman telephones the hospital to report that she is in labor, what guidance should the nurse provide or information should the nurse obtain? a. Tell the woman to stay home until her membranes rupture. b. Emphasize that food and fluid intake should stop. c. Arrange for the woman to come to the hospital for labor evaluation. d. Ask the woman to describe why she believes she is in labor.

ANS: D - Ask the woman to describe why she believes she is in labor. Assessment begins at the first contact with the woman, whether by telephone or in person. By asking the woman to describe her signs and symptoms, the nurse can begin her assessment and gather data. The initial nursing activity should be to gather data about the woman's status. The amniotic membranes may or may not spontaneously rupture during labor. The client may be instructed to stay home until the uterine contractions become strong and regular. Before instructing the woman to come to the hospital, the nurse should initiate her assessment during the telephone interview. After this assessment has been made, the nurse may want to discuss the appropriate oral intake for early labor, such as light foods or clear liquids, depending on the preference of the client or her primary health care provider

The obstetric nurse is preparing the client for an emergency cesarean birth, with no time to administer spinal anesthesia. The nurse is aware of and prepared for the greatest risk of administering general anesthesia to the client. What is this risk? a. Respiratory depression b. Uterine relaxation c. Inadequate muscle relaxation d. Aspiration of stomach contents

ANS: D - Aspiration of stomach contents Aspiration of acidic gastric contents with possible airway obstruction is a potentially fatal complication of general anesthesia. Respirations can be altered during general anesthesia, and the anesthesiologist will take precautions to maintain proper oxygenation. Uterine relaxation can occur with some anesthesia but can be monitored and prevented. Inadequate muscle relaxation can be improved with medication.

A multiparous woman has been in labor for 8 hours. Her membranes have just ruptured. What is the nurse's highest priority in this situation? a. Prepare the woman for imminent birth. b. Notify the woman's primary health care provider. c. Document the characteristics of the fluid. d. Assess the fetal heart rate (FHR) and pattern.

ANS: D - Assess the fetal heart rate (FHR) and pattern. The umbilical cord may prolapse when the membranes rupture. The FHR and pattern should be closely monitored for several minutes immediately after the ROM to ascertain fetal well-being, and the findings should be documented. The ROM may increase the intensity and frequency of the uterine contractions, but it does not indicate that birth is imminent. The nurse may notify the primary health care provider after ROM occurs and the fetal well-being and response to ROM have been assessed. The nurse's priority is to assess fetal well-being. The nurse should document the characteristics of the amniotic fluid, but the initial response is to assess fetal well-being and the response to ROM.

A woman who is pregnant for the first time is dilated 3 cm and having contractions every 5 minutes. She is groaning and perspiring excessively; she states that she did not attend childbirth classes. What is the optimal intervention for the nurse to provide at this time? a. Notify the woman's health care provider. b. Administer the prescribed narcotic analgesic. c. Assure her that her labor will be over soon. d. Assist her with simple breathing and relaxation instructions.

ANS: D - Assist her with simple breathing and relaxation instructions. By reducing tension and stress, both focusing and relaxation techniques will allow the woman in labor to rest and conserve energy for the task of giving birth. For those who have had no preparation, instruction in simple breathing and relaxation can be given in early labor and is often successful. The nurse can independently perform many functions in labor and birth, such as teaching and support. Pain medication may be an option for this client. However, the initial response of the nurse should include teaching the client about her options. The length of labor varies among individuals, but the first stage of labor is the longest. At 3 cm of dilation with contractions every 5 minutes, this woman has a significant amount of labor yet to experience.

A client states that she does not drink milk. Which foods should the nurse encourage this woman to consume in greater amounts to increase her calcium intake? a. Fresh apricots b. Canned clams c. Spaghetti with meat sauce d. Canned sardines

ANS: D - Canned sardines Sardines are rich in calcium. Fresh apricots, canned clams, and spaghetti with meat sauce are not high in calcium.

Which structure is responsible for oxygen and carbon dioxide transport to and from the maternal bloodstream? a. Decidua basalis b. Blastocyst c. Germ layer d. Chorionic villi

ANS: D - Chorionic villi Chorionic villi are fingerlike projections that develop out of the trophoblast and extend into the blood-filled spaces of the endometrium. The villi obtain oxygen and nutrients from the maternal bloodstream and dispose carbon dioxide and waste products into the maternal blood. The decidua basalis is the portion of the decidua (endometrium) under the blastocyst where the villi attach. The blastocyst is the embryonic development stage after the morula; implantation occurs at this stage. The germ layer is a layer of the blastocyst.

The major source of nutrients in the diet of a pregnant woman should be composed of what? a. Simple sugars b. Fats c. Fiber d. Complex carbohydrates

ANS: D - Complex carbohydrates Complex carbohydrates supply the pregnant woman with vitamins, minerals, and fiber. The most common simple carbohydrate is table sugar, which is a source of energy but does not provide any nutrients. Fats provide 9 kcal in each gram, in contrast to carbohydrates and proteins, which provide only 4 kcal in each gram. Fiber is primarily supplied by complex carbohydrates

To reassure and educate their pregnant clients regarding changes in their blood pressure, nurses should be cognizant of what? a. A blood pressure cuff that is too small produces a reading that is too low; a cuff that is too large produces a reading that is too high. b. Shifting the client's position and changing from arm to arm for different measurements produces the most accurate composite blood pressure reading at each visit. c. Systolic blood pressure slightly increases as the pregnancy advances; diastolic pressure remains constant. d. Compression of the iliac veins and inferior vena cava by the uterus contributes to hemorrhoids in the later stage of a term pregnancy.

ANS: D - Compression of the iliac veins and inferior vena cava by the uterus contributes to hemorrhoids in the later stage of a term pregnancy. Compression of the iliac veins and inferior vena cava by the uterus contributes to hemorrhoids in the later stage of a term pregnancy. This compression also leads to varicose veins in the legs and vulva. The tightness of a blood pressure cuff that is too small produces a reading that is too high; similarly, the looseness of a cuff that is too large results in a reading that is too low. Because maternal positioning affects readings, blood pressure measurements should be obtained in the same arm and with the woman in the same position. The systolic blood pressure generally remains constant but may decline slightly as the pregnancy advances. The diastolic blood pressure first decreases and then gradually increases

A primigravida at 39 weeks of gestation is observed for 2 hours in the intrapartum unit. The FHR has been normal. Contractions are 5 to 9 minutes apart, 20 to 30 seconds in duration, and of mild intensity. Cervical dilation is 1 to 2 cm and uneffaced (unchanged from admission). Membranes are intact. What disposition would the nurse anticipate? a. Admitted and prepared for a cesarean birth b. Admitted for extended observation c. Discharged home with a sedative d. Discharged home to await the onset of true labor

ANS: D - Discharged home to await the onset of true labor This situation describes a woman with normal assessments who is probably in false labor and will likely not deliver rapidly once true labor begins. No further assessments or observations are indicated; therefore, the client will be discharged along with instructions to return when contractions increase in intensity and frequency. Neither a cesarean birth nor a sedative is required at this time.

The nurse performs a vaginal examination to assess a client's labor progress. Which action should the nurse take next? a. Perform an examination at least once every hour during the active phase of labor. b. Perform the examination with the woman in the supine position. c. Wear two clean gloves for each examination. d. Discuss the findings with the woman and her partner.

ANS: D - Discuss the findings with the woman and her partner. The nurse should discuss the findings of the vaginal examination with the woman and her partner, as well as report the findings to the primary care provider. A vaginal examination should be performed only when indicated by the status of the woman and her fetus. The woman should be positioned so as to avoid supine hypotension. The examiner should wear a sterile glove while performing a vaginal examination for a laboring woman

A woman in the 34th week of pregnancy reports that she is very uncomfortable because of heartburn. Which recommendation would be appropriate for this client? a. Substitute other calcium sources for milk in her diet. b. Lie down after each meal. c. Reduce the amount of fiber she consumes. d. Eat five small meals daily.

ANS: D - Eat five small meals daily. Eating small, frequent meals may help with heartburn, nausea, and vomiting. Substituting other calcium sources for milk, lying down after eating, and reducing fiber intake are inappropriate dietary suggestions for all pregnant women and do not alleviate heartburn.

Nutrition is an alterable and important preventive measure for a variety of potential problems such as low birth weight and prematurity. While completing the physical assessment of the pregnant client, the nurse is able to evaluate the client's nutritional status by observing a number of physical signs. Which physical sign indicates to the nurse that the client has unmet nutritional needs? a. Normal heart rate, rhythm, and blood pressure b. Bright, clear, and shiny eyes c. Alert and responsive with good endurance d. Edema, tender calves, and tingling

ANS: D - Edema, tender calves, and tingling The physiologic changes of pregnancy may complicate the interpretation of physical findings. Lower extremity edema often occurs when caloric and protein deficiencies are present; however, edema in the lower extremities may also be a common physical finding during the third trimester. Completing a thorough health history and physical assessment and requesting further laboratory testing, if indicated, are essential for the nurse. The malnourished pregnant client may display rapid heart rate, abnormal rhythm, enlarged heart, and elevated blood pressure. A client receiving adequate nutrition will have bright, shiny eyes with no sores and moist, pink membranes. Pale or red membranes, dryness, infection, dull appearance of the cornea, or blue sclerae are signs of poor nutrition. A client who is alert and responsive with good endurance is well nourished. A listless, cachectic, easily fatigued, and tired presentation would be an indication of a poor nutritional status.

Maternity nurses often have to answer questions about the many, sometimes unusual, ways people have tried to make the birthing experience more comfortable. Which information regarding nonpharmacologic pain relief is accurate? a. Music supplied by the support person has to be discouraged because it could disturb others or upset the hospital routine. b. Women in labor can benefit from sitting in a bathtub, but they must limit immersion to no longer than 15 minutes at a time. c. Effleurage is permissible, but counterpressure is almost always counterproductive. d. Electrodes attached to either side of the spine to provide high-intensity electrical impulses facilitate the release of endorphins.

ANS: D - Electrodes attached to either side of the spine to provide high-intensity electrical impulses facilitate the release of endorphins. Transcutaneous electrical nerve stimulation (TENS) may help and is most useful for lower back pain that occurs during the first stage of labor. Music may be very helpful for reducing tension and certainly can be accommodated by the hospital. Women can stay in a bath as long as they want, although repeated baths with breaks might be more effective than one long bath. Counterpressure can help the woman cope with lower back pain.

Which statement regarding the care of a client in labor is correct and important to the nurse as he or she formulates the plan of care? a. The woman's blood pressure will increase during contractions and fall back to prelabor normal levels between contractions. b. The use of the Valsalva maneuver is encouraged during the second stage of labor to relieve fetal hypoxia. c. Having the woman point her toes will reduce leg cramps. d. Endogenous endorphins released during labor will raise the woman's pain threshold and produce sedation.

ANS: D - Endogenous endorphins released during labor will raise the woman's pain threshold and produce sedation. The endogenous endorphins released during labor will raise the woman's pain threshold and produce sedation. In addition, physiologic anesthesia of the perineal tissues, caused by the pressure of the presenting part, decreases the mother's perception of pain. Blood pressure levels increase during contractions but remain somewhat elevated between them. The use of the Valsalva maneuver is discouraged during the second stage labor because of a number of unhealthy outcomes, including fetal hypoxia. Pointing the toes can cause leg cramps, as can the process of labor itself.

What physiologic change occurs as the result of increasing the infusion rate of nonadditive IV fluids? a. Maintaining normal maternal temperature b. Preventing normal maternal hypoglycemia c. Increasing the oxygen-carrying capacity of the maternal blood d. Expanding maternal blood volume

ANS: D - Expanding maternal blood volume Filling the mother's vascular system increases the amount of blood available to perfuse the placenta and may correct hypotension. Increasing fluid volume may alter the maternal temperature only if she is dehydrated. Most IV fluids for laboring women are isotonic and do not provide extra glucose. Oxygen-carrying capacity is increased by adding more red blood cells.

Who is most likely to experience the phenomenon of someone other than the mother-to-be having pregnancy-like symptoms such as nausea and weight gain? a. Mother of the pregnant woman b. Couple's teenage daughter c. Sister of the pregnant woman d. Expectant father

ANS: D - Expectant father An expectant father's experiencing of his partner's pregnancy-like symptoms is called the couvade syndrome. The mother of the pregnant woman is unlikely to experience this phenomenon. She may be excited about becoming a grandmother or see her daughter's pregnancy as a reminder that she is getting old. A couple's teenage daughter is usually preoccupied with her own sexual development and may have difficulty accepting the overwhelming evidence of her parents' sexual activity. It is the father of the pregnant woman, not the sister, who experiences these symptoms.

Certain changes stimulate chemoreceptors in the aorta and carotid bodies to prepare the fetus for initiating respirations immediately after birth. Which change in fetal physiologic activity is not part of this process? a. Fetal lung fluid is cleared from the air passages during labor and vaginal birth. b. Fetal partial pressure of oxygen (PO2) decreases. c. Fetal partial pressure of carbon dioxide in arterial blood (PaCO2) increases. d. Fetal respiratory movements increase during labor.

ANS: D - Fetal respiratory movements increase during labor. Fetal respiratory movements actually decrease during labor. Fetal lung fluid is cleared from the air passages during labor and vaginal birth. Fetal PO2 decreases, and fetal PaCO2 increases.

Which collection of risk factors will most likely result in damaging lacerations, including episiotomies? a. Dark-skinned woman who has had more than one pregnancy, who is going through prolonged second-stage labor, and who is attended by a midwife b. Reddish-haired mother of two who is going through a breech birth c. Dark-skinned first-time mother who is going through a long labor d. First-time mother with reddish hair whose rapid labor was overseen by an obstetrician

ANS: D - First-time mother with reddish hair whose rapid labor was overseen by an obstetrician Reddish-haired women have tissue that is less distensible than darker-skinned women and therefore may have less efficient healing. First-time mothers are also at greater risk, especially with breech births, long second-stage labors, or rapid labors during which the time for the perineum to stretch is insufficient. The rate of episiotomies is higher when obstetricians rather than midwives attend the births. The woman in the first scenario (a) is at low risk for either damaging lacerations or an episiotomy. She is multiparous, has dark skin, and is being attended by a midwife, who is less likely to perform an episiotomy. Reddish-haired women have tissue that is less distensible than that of darker-skinned women. Consequently, the client in the second scenario (b) is at increased risk for lacerations; however, she has had two previous deliveries, which result in a lower likelihood of an episiotomy. The fact that the woman in the third scenario (c) is experiencing a prolonged labor might increase her risk for lacerations. Fortunately, she is dark skinned, which indicates that her tissue is more distensible than that of fair-skinned women and therefore less susceptible to injury.

While assessing the vital signs of a pregnant woman in her third trimester, the client complains of feeling faint, dizzy, and agitated. Which nursing intervention is appropriate? a. Have the patient stand up, and then retake her BP. b. Have the patient sit down, and then hold her arm in a dependent position. c. Have the patient lie supine for 5 minutes, and then recheck her BP on both arms. d. Have the patient turn to her left side, and then recheck her BP in 5 minutes.

ANS: D - Have the patient turn to her left side, and then recheck her BP in 5 minutes. BP is affected by maternal position during pregnancy. The supine position may cause occlusion of the vena cava and descending aorta. Turning the pregnant woman to a lateral recumbent position alleviates pressure on the blood vessels and quickly corrects supine hypotension. Pressures are significantly higher when the client is standing. This option causes an increase in systolic and diastolic pressures. The arm should be supported at the same level of the heart. The supine position may cause occlusion of the vena cava and descending aorta, creating hypotension

A woman in the active phase of the first stage of labor is using a shallow pattern of breathing, which is approximately twice the normal adult breathing rate. She starts to complain about feeling lightheaded and dizzy and states that her fingers are tingling. Which intervention should the nurse immediately initiate? a. Contact the woman's physician. b. Tell the woman to slow her pace of her breathing. c. Administer oxygen via a mask or nasal cannula. d. Help her breathe into a paper bag.

ANS: D - Help her breathe into a paper bag. This woman is experiencing the side effects of hyperventilation, which include the symptoms of lightheadedness, dizziness, tingling of the fingers, or circumoral numbness. Having the woman breathe into a paper bag held tightly around her mouth and nose may eliminate respiratory alkalosis and enable her to rebreathe carbon dioxide and replace the bicarbonate ion.

hCG is an important biochemical marker for pregnancy and therefore the basis for many tests. Which statement regarding hCG is true? a. hCG can be detected as early as weeks after conception. b. hCG levels gradually and uniformly increase throughout pregnancy. c. Significantly lower-than-normal increases in the levels of hCG may indicate a postdate pregnancy. d. Higher-than-normal levels of hCG may indicate an ectopic pregnancy or Down syndrome.

ANS: D - Higher-than-normal levels of hCG may indicate an ectopic pregnancy or Down syndrome. Higher hCG levels also could be a sign of a multiple gestation. hCG can be detected as early as 7 to 10 days after conception. The hCG levels fluctuate during pregnancy, peaking, declining, stabilizing, and then increasing again. Abnormally slow increases may indicate impending miscarriage.

The perinatal nurse realizes that an FHR that is tachycardic, bradycardic, has late decelerations, or loss of variability is nonreassuring and is associated with which condition? a. Hypotension b. Cord compression c. Maternal drug use d. Hypoxemia

ANS: D - Hypoxemia Nonreassuring FHR patterns are associated with fetal hypoxemia. Fetal bradycardia may be associated with maternal hypotension. Variable FHR decelerations are associated with cord compression. Maternal drug use is associated with fetal tachycardia

Which action is the first priority for the nurse who is assessing the influence of culture on a client's diet? a. Evaluate the client's weight gain during pregnancy. b. Assess the socioeconomic status of the client. c. Discuss the four food groups with the client. d. Identify the food preferences and methods of food preparation common to the client's culture.

ANS: D - Identify the food preferences and methods of food preparation common to the client's culture. Understanding the client's food preferences and how she prepares food will assist the nurse in determining whether the client's culture is adversely affecting her nutritional intake. An evaluation of a client's weight gain during pregnancy should be included for all clients, not only for clients from different cultural backgrounds. The socioeconomic status of the client may alter the nutritional intake but not the cultural influence. Teaching the food groups to the client should come after assessing her food preferences.

The nurse is evaluating the EFM tracing of the client who is in active labor. Suddenly, the FHR drops from its baseline of 125 down to 80 beats per minute. The mother is repositioned, and the nurse provides oxygen, increased IV fluids, and performs a vaginal examination. The cervix has not changed. Five minutes have passed, and the FHR remains in the 80s. What additional nursing measures should the nurse take next? a. Call for help. b. Insert a Foley catheter. c. Start administering Pitocin. d. Immediately notify the care provider.

ANS: D - Immediately notify the care provider. To relieve an FHR deceleration, the nurse can reposition the mother, increase IV fluids, and provide oxygen. If oxytocin is infusing, then it should be discontinued. If the FHR does not resolve, then the primary care provider should be immediately notified. Inserting a Foley catheter is an inappropriate nursing action. If the FHR were to continue in a nonreassuring pattern, then a cesarean section could be warranted, which would require a Foley catheter. However, the physician must make that determination. The administration of Pitocin may place additional stress on the fetus

What is the most basic information that a nurse should be able to share with a client who asks about the process of conception? a. Ova are considered fertile 48 to 72 hours after ovulation. b. Sperm remain viable in the woman's reproductive system for an average of 12 to 24 hours. c. Conception is achieved when a sperm successfully penetrates the membrane surrounding the ovum. d. Implantation in the endometrium occurs 6 to 10 days after conception.

ANS: D - Implantation in the endometrium occurs 6 to 10 days after conception. After implantation, the endometrium is called the decidua. Ova are considered fertile for approximately 24 hours after ovulation. Sperm remain viable in the woman's reproductive system for an average of 2 to 3 days. Penetration of the ovum by the sperm is called fertilization. Conception occurs when the zygote, the first cell of the new individual, is formed.

Pregnancy hormones prepare the vagina for stretching during labor and birth. Which change related to the pelvic viscera should the nurse share with the client? a. Because of a number of changes in the cervix, abnormal Papanicolaou (Pap) tests are easier to evaluate. b. Quickening is a technique of palpating the fetus to engage it in passive movement. c. The deepening color of the vaginal mucosa and cervix (Chadwick sign) usually appears in the second trimester or later as the vagina prepares to stretch during labor. d. Increased vascularity of the vagina increases sensitivity and may lead to a high degree of arousal, especially in the second trimester.

ANS: D - Increased vascularity of the vagina increases sensitivity and may lead to a high degree of arousal, especially in the second trimester. Increased sensitivity and an increased interest in sex sometimes go together and frequently occur during the second trimester. These cervical changes make evaluation of abnormal Pap tests more difficult. Quickening is the first recognition of fetal movements by the mother. Ballottement is a technique used to palpate the fetus. The Chadwick sign appears from the 6 to 8 weeks of gestation.

Which FHR finding is the most concerning to the nurse who is providing care to a laboring client? a. Accelerations with fetal movement b. Early decelerations c. Average FHR of 126 beats per minute d. Late decelerations

ANS: D - Late decelerations Late decelerations are caused by uteroplacental insufficiency and are associated with fetal hypoxemia. Late FHR decelerations are considered ominous if they are persistent and left uncorrected. Accelerations with fetal movement are an indication of fetal well-being. Early decelerations in the FHR are associated with head compression as the fetus descends into the maternal pelvic outlet; they are not generally a concern during normal labor

Which statement correctly describes the effects of various pain factors? a. Higher prostaglandin levels arising from dysmenorrhea can blunt the pain of childbirth. b. Upright positions in labor increase the pain factor because they cause greater fatigue. c. Women who move around trying different positions experience more pain. d. Levels of pain-mitigating beta-endorphins are higher during a spontaneous, natural childbirth.

ANS: D - Levels of pain-mitigating beta-endorphins are higher during a spontaneous, natural childbirth. Higher endorphin levels help women tolerate pain and reduce anxiety and irritability. Higher prostaglandin levels correspond to more severe labor pains. Upright positions in labor usually result in improved comfort and less pain. Moving freely to find more comfortable positions is important for reducing pain and muscle tension.

Which gastrointestinal alteration of pregnancy is a normal finding? a. Insufficient salivation (ptyalism) is caused by increases in estrogen. b. Acid indigestion (pyrosis) begins early but declines throughout pregnancy. c. Hyperthyroidism often develops (temporarily) because hormone production increases. d. Nausea and vomiting rarely have harmful effects on the fetus and may be beneficial.

ANS: D - Nausea and vomiting rarely have harmful effects on the fetus and may be beneficial. Normal nausea and vomiting rarely produce harmful effects and may be less likely to result in miscarriage or preterm labor. Ptyalism is excessive salivation that may be caused by a decrease in unconscious swallowing or by stimulation of the salivary glands. Pyrosis begins as early as the first trimester and intensifies through the third trimester. Increased hormone production does not lead to hyperthyroidism in pregnant women.

The Valsalva maneuver can be described as the process of making a forceful bearing-down attempt while holding one's breath with a closed glottis and a tightening of the abdominal muscles. When is it appropriate to instruct the client to use this maneuver? a. During the second stage to enhance the movement of the fetus b. During the third stage to help expel the placenta c. During the fourth stage to expel blood clots d. Not at all

ANS: D - Not at all The client should not be instructed to use this maneuver. This process stimulates the parasympathetic division of the autonomic nervous system and produces a vagal response (decrease in heart rate and blood pressure.) An alternative method includes instructing the client to perform open-mouth and open-glottis breathing and pushing

A woman who is 16 weeks pregnant has come in for a follow-up visit with her significant other. To reassure the client regarding fetal well-being, which is the highest priority action for the nurse to perform? a. Assess the fetal heart tones with a Doppler stethoscope. b. Measure the girth of the woman's abdomen. c. Complete an ultrasound examination (sonogram). d. Offer the woman and her family the opportunity to listen to the fetal heart tones.

ANS: D - Offer the woman and her family the opportunity to listen to the fetal heart tones. To provide the parents with the greatest sense of reassurance, the nurse should offer to have the client and her significant other the chance to listen to their baby's heartbeat. A fetoscope can detect the fetal heart rate around 20 weeks of gestation. Doppler can detect the fetal heart rate between 10 and 12 weeks and should be performed as part of routine fetal assessment. Abdominal girth is not a valid measure for determining fetal well-being. Fundal height is an important measure that should be determined with precision, with the same technique and positioning of the client consistently used at every prenatal visit. Routine ultrasound examinations are recommended in early pregnancy; they date the pregnancy and provide useful information about the health of the fetus. However, they are not necessary at each prenatal visit.

Which basic type of pelvis includes the correct description and percentage of occurrence in women? a. Gynecoid: classic female pelvis; heart shaped; 75% b. Android: resembling the male pelvis; wide oval; 15% c. Anthropoid: resembling the pelvis of the ape; narrow; 10% d. Platypelloid: flattened, wide, and shallow pelvis; 3%

ANS: D - Platypelloid: flattened, wide, and shallow pelvis; 3% A platypelloid pelvis is flattened, wide, and shallow; approximately 3% of women have this shape. The gynecoid pelvis is the classic female shape, slightly ovoid and rounded; approximately 50% of women have this shape. An android or malelike pelvis is heart shaped; approximately 23% of women have this shape. An anthropoid or apelike pelvis is oval and wide; approximately 24% of women have this shape.

Which hormone is essential for maintaining pregnancy? a. Estrogen b. hCG c. Oxytocin d. Progesterone

ANS: D - Progesterone Progesterone is essential for maintaining pregnancy; it does so by relaxing smooth muscles, which reduces uterine activity and prevents miscarriage. Estrogen plays a vital role in pregnancy, but it is not the primary hormone for maintaining pregnancy. hCG levels rise at implantation but decline after 60 to 70 days. Oxytocin stimulates uterine contractions.

An 18-year-old pregnant woman, gravida 1, para 0, is admitted to the labor and birth unit with moderate contractions every 5 minutes that last 40 seconds. The client states, "My contractions are so strong, I don't know what to do." Before making a plan of care, what should the nurse's first action be? a. Assess for fetal well-being. b. Encourage the woman to lie on her side. c. Disturb the woman as little as possible. d. Recognize that pain is personalized for each individual.

ANS: D - Recognize that pain is personalized for each individual. Each woman's pain during childbirth is unique and is influenced by a variety of physiologic, psychosocial, and environmental factors. A critical issue for the nurse is how support can make a difference in the pain of the woman during labor and birth. This scenario includes no information that would indicate fetal distress or a logical reason to be overly concerned about the well-being of the fetus. The left lateral position is used to alleviate fetal distress, not maternal stress. The nurse has an obligation to provide physical, emotional, and psychosocial care and support to the laboring woman. This client clearly needs support.

Numerous changes in the integumentary system occur during pregnancy. Which change persists after birth? a. Epulis b. Chloasma c. Telangiectasia d. Striae gravidarum

ANS: D - Striae gravidarum Striae gravidarum, or stretch marks, reflect a separation within the underlying connective tissue of the skin. They usually fade after birth, although they never completely disappear. An epulis is a red, raised nodule on the gums that easily bleeds; it disappears or shrinks after giving birth. Chloasma, or the mask of pregnancy, is a blotchy, brown hyperpigmentation of the skin over the cheeks, nose, and forehead, especially in dark-complexioned pregnant women. Chloasma usually fades after the birth. Telangiectasia, or vascular spiders, are tiny, star-shaped or branchlike, slightly raised, pulsating end-arterioles usually found on the neck, thorax, face, and arms. They occur as a result of elevated levels of circulating estrogen and usually disappear after birth.

A pregnant woman is at 38 weeks of gestation. She wants to know whether there are any signs that "labor is getting close to starting." Which finding is an indication that labor may begin soon? a. Weight gain of 1.5 to 2 kg (3 to 4 lb) b. Increase in fundal height c. Urinary retention d. Surge of energy

ANS: D - Surge of energy Women speak of having a burst of energy before labor. The woman may lose 0.5 to 1.5 kg, as a result of water loss caused by electrolyte shifts that, in turn, are caused by changes in the estrogen and progesterone levels. When the fetus descends into the true pelvis (called lightening), the fundal height may decrease. Urinary frequency may return before labor.

In which situation would the nurse be called on to stimulate the fetal scalp? a. As part of fetal scalp blood sampling b. In response to tocolysis c. In preparation for fetal oxygen saturation monitoring d. To elicit an acceleration in the FHR

ANS: D - To elicit an acceleration in the FHR The scalp can be stimulated using digital pressure during a vaginal examination. Fetal scalp blood sampling involves swabbing the scalp with disinfectant before a sample is collected. The nurse stimulates the fetal scalp to elicit an acceleration of the FHR. Tocolysis is relaxation of the uterus. Fetal oxygen saturation monitoring involves the insertion of a sensor.

In her work with pregnant women of different cultures, a nurse practitioner has observed various practices that seemed unfamiliar. The nurse practitioner has learned that cultural rituals and practices during pregnancy seem to have one purpose in common. Which statement best describes that purpose? a. To promote family unity b. To ward off the "evil eye" c. To appease the gods of fertility d. To protect the mother and fetus during pregnancy

ANS: D - To protect the mother and fetus during pregnancy Although many cultures consider pregnancy normal, certain practices are expected of women of all cultures to ensure a good outcome. Cultural prescriptions tell women what to do, and cultural proscriptions establish taboos. The purposes of these practices are to prevent maternal illness resulting from a pregnancy-induced imbalanced state and to protect the vulnerable fetus. Promoting family unity is important, although not usually the premise for cultural rituals and practices. Warding off the "evil eye" may be specific to one particular culture; however, it is not the primary purpose of these practices. Appeasing the gods of fertility is not the impetus behind cultural rituals.

The nurse is using intermittent auscultation (IA) to locate the fetal heartbeat. Which statement regarding this method of surveillance is accurate? a. The nurse can be expected to cover only two or three clients when IA is the primary method of fetal assessment. b. The best course is to use the descriptive terms associated with EFM when documenting results. c. If the heartbeat cannot be immediately found, then a shift must be made to EFM. d. Ultrasound can be used to find the FHR and to reassure the mother if the initial difficulty is a factor.

ANS: D - Ultrasound can be used to find the FHR and to reassure the mother if the initial difficulty is a factor. Locating fetal heartbeats often takes time. Mothers can be verbally reassured and reassured by viewing the ultrasound pictures if that device is used to help locate the heartbeat. When used as the primary method of fetal assessment, IA requires a nurse-to-client ratio of one to one. Documentation should use only terms that can be numerically defined; the usual visual descriptions of EFM are inappropriate.

Which sign or symptom is considered a first-trimester warning sign and should be immediately reported by the pregnant woman to her health care provider? a. Nausea with occasional vomiting b. Fatigue c. Urinary frequency d. Vaginal bleeding

ANS: D - Vaginal bleeding Signs and symptoms that must be reported include severe vomiting, fever and chills, burning on urination, diarrhea, abdominal cramping, and vaginal bleeding. These symptoms may be signs of complications of the pregnancy. Nausea with occasional vomiting is a normal first-trimester complaint. Although it may be worrisome or annoying to the mother, it is not usually an indication of a problem with the pregnancy. Fatigue is common during the first trimester. Because of physiologic changes that happen during pregnancy, clients should be taught that urinary frequency is normal.

Which vitamins or minerals may lead to congenital malformations of the fetus if taken in excess by the mother? a. Zinc b. Vitamin D c. Folic acid d. Vitamin A

ANS: D - Vitamin A If taken in excess, vitamin A causes a number of problems. An analog of vitamin A appears in prescribed acne medications, which must not be taken during pregnancy. Zinc, vitamin D, and folic acid are all vital to good maternity and fetal health and are highly unlikely to be consumed in excess.

Which pregnant woman should strictly follow weight gain recommendations during pregnancy? a. Pregnant with twins b. In early adolescence c. Shorter than 62 inches or 157 cm d. Was 20 pounds overweight before pregnancy

ANS: D - Was 20 pounds overweight before pregnancy A weight gain of 5 to 9 kg will provide sufficient nutrients for the fetus. Overweight and obese women should be advised to lose weight before conception to achieve the best pregnancy outcomes. A higher weight gain in twin gestations may help prevent low birth weights. Adolescents need to gain weight toward the higher acceptable range, which provides for their own growth, as well as for fetal growth. In the past, women of short stature were advised to restrict their weight gain; however, evidence to support these guidelines has not been found.

Many pregnant women have questions regarding work and travel during pregnancy. Which education is a priority for the nurse to provide? a. Women should sit for as long as possible and cross their legs at the knees from time to time for exercise. b. Women should avoid seat belts and shoulder restraints in the car because they press on the fetus. c. Metal detectors at airport security checkpoints can harm the fetus if the woman passes through them a number of times. d. While working or traveling in a car or on an airplane, women should arrange to walk around at least every hour or so.

ANS: D - While working or traveling in a car or on an airplane, women should arrange to walk around at least every hour or so. Periodic walking helps prevent thrombophlebitis. Pregnant women should avoid sitting or standing for long periods and crossing the legs at the knees. Pregnant women must wear lap belts and shoulder restraints. The most common injury to the fetus comes from injury to the mother. Metal detectors at airport security checkpoints do not harm fetuses.

A woman arrives at the clinic for a pregnancy test. The first day of her LMP was September 10, 2014. Her expected date of birth (EDB) is __________.

ANS: June 17, 2015


Kaugnay na mga set ng pag-aaral

PC Pro CHapter 3 (3.8.3.14) Questions

View Set

Lifespan Development: Chapter One

View Set

Ch10 - Collecting Data by Observation

View Set

BIOL&160 - SmartBook Assignment Chapter 11. DNA Technology

View Set

2.12 One-Variable Linear Equations and Inequalities; Unit Exam - Algebra

View Set

Chapter 9: Production and Operations Management

View Set